You are on page 1of 78

Two different two-digit natural numbers are written beside each other such that the larger number

is written on the left. When the absolute difference of the two numbers is subtracted from the fourdigit number so formed, the number obtained is 5481. What is the sum of the two two-digit numbers? Given abcd - (ab -cd ) = 5481 1000a +100b + 10c + d - 10a+10c-b+d = 5481 990a + 99b + 20c + 2d = 5481 Clearly a=5 , so 990 *5 = 4950 so we left with 5481 - 4950 = 531 99*5 = 495 , so b = 5 ; 531 - 495 = 36 hence c=1 and d=8 . so number is 5518 . (ab>cd) And sum = 55 +18 = 73 (ans) A natural number x leaves a remainder 1 when divided by p. The resultant quotient, when divided by q, leaves a remainder 2. The resultant quotient, when divided by r, leaves a remainder 3 and the quotient, thus obtained, is exactly divisible by 5. If p, q and r are all natural numbers, which of the following is the least possible value of x? Number must be of the form ( p ( q ( r ( 5k )+ 3 )+ 2 )+ 1 Now, putting the values of p=2,q=3,r=4 & k = 0 As the number needs to be smallest, we must take as small values as possible. Now, remainder with p is 1, so p >1. So, p = 2 rem with q = 2, so, q>2, so q = 3 rem with r = 3, so, r > 3, so, r = 4 One of the smaller sides of a right angled triangle is (2^2)*(3^3)*(4^4)*(5^5)*(6^6)*(7^7) . It is known that other two sides are integers.How many triangles of this type are possible. b^2 = (2^4)*(3^6)*(4^8 )*(5^10)*(6^12)*(7^14) = (2^32)*(3^18 )*(5^10)*(7^14) => (a + c)(a - c) = (2^32)*(3^18 )*(5^10)*(7^14) Since both (a + c) and (a - c) are even, say 2k and 2n, then k*n = (2^30)*(3^18 )*(5^10)*(7^14) Now, k is greater than n, so we just have to write (2^30)*(3^18 )*(5^10)*(7^14) as product of two numbers where k is the greater one and n is smaller one, then we can get values for a and c So, answer will be (31*19*11*15 - 1)/2 = 48592 a + b > c means a^2 + b^2 + 2ab > c^2 We have taken a^2 + b^2 = c^2 hence a^2 + b^2 + 2ab > c^2 (fulfilled) Also for a - b < c a^2 + b^2 = c^2 => a^2 + b^2 - 2ab < c^2 (again fulfilled)

its like if (a - c) is even then (a + c) will also be even for any integers a and c. If (a - c) is odd, then (a + c) will also be odd. So, if (a - c)(a + c) = even, then both should be even Suppsoe (a - c) = even and (a + c ) = odd, then 2a = even + odd => a is not an integer. there is a sheet of paper. A picks it up and tears it into 10 pieces. B comes and picks up atleast one of the 10 pieces and tears each of them into another 10 pieces. C comes and does the same thing. Which of these can never be the number of pieces of paper in the end of the process? a) 649 b) 469 c) 1000 d) 759 10*10+0 = 100 10*9+1 = 91 10*8+2 =82 .... no of pieces is of the form 9n+1 In a test consisting of 15 questions, 3 marks are awarded for a correct answer, 1 mark is deducted for an incorrect answer and no mark is awarded for an unattempted question. If a student attempts at least one question in the paper, what is the number of distinct scores that he can get X+y+z =15 3X-y Maximum marks = 15*3 = 45 Minimum marks = -15 So, we have 61 possibilities. But, we cannot make marks = 44, 43 and 40. So total =58 Basically if no of marks are +n for correct and -1 for incorrect, then Number of score which are not possible is given by (n - 1) + (n - 2) + ... + 1 = n(n - 1)/2 when one gets 30 correct; he gets 180. And with 29 correct, he gets 174. So, for a score more that 174, one needs 30 correct. And with 30 correct i.e. all correct, you can have 0 wrong. So, 30 correct, 1 wrong or 2 wrong or ..upto 5 wrong is not possible. With 29 correct, 1 wrong is possible. But 2, 3 ,4 or 5 is not. With 28 correct, 3, 4, 5 wrong is not possible. So, basically, when we have +6 and -1: Number of invalid scores are: 1+2+3+4+5 = 15 When we have +5 and -1: Number of invalid scores are: 1+2+3+4 = 10 When we have +4 and -1: Number of invalid scores are: 1+2+3 = 6 What is the H.C.F. of (2^100-1) and (2^120-1) H.C.F of power is 20 so 2^20-1

It takes 6 days for 3 women and 2 men working together to complete a work. Three men would do the same work five days sooner than nine women .How many times does the output of a man exeed that of woman...? A certain bacteria reproduces itself at the rate of 10% every min. In how much time will it double itself ? if after 't' time a certain rate doubles itself at 'r%' ci, then.. r*t=72.. so ur ans is 7.2.. similar goes for quadraple...r*t=144 and so on.... There are 10 heaps of coins, consisting of 10 coins each. every coin looks the same. there are 9 heaps consisting of coins of same weight, the other heap has coins of different weight. the coins look the same to the human eye, and there is no way one can tell the difference in weights of the coin by just looking at it. If ur allowed to weigh the coin(s) only once. then how wld u knw which heap consists of the coins with a different weight? (u hv an electronic weighing machine, works only once n then selfdestructs). take coins in following fashion : 10th heap = 10 coins 9th heap = 9 coins 8th heap = 8 coins . . . . 1st heap = 1 coin say correct weight = 1 gram and false weight = 2 gram now total weight should be = 55 gram so,if 20 gram extra then false heap is 10th if 18 gram extra then its 9th heap and so on You are selecting 10 nos randomely out of a first 100 odd numbers. Sum of these 10 odd numbers is A. How many diff values of A are possible?? A. C(100, 10) B. 1801 C. 1800 D. 901 We can not select C(100,10) because lets take a case when we have to select 4 odd number when sum is different. 3 5 7 9 =24 1 7 5 11 =24 So approach : Sum of 10 odd numbers will always be even

Minimum possible sum = 1 + 3 + ... + 19 = 100 Maximum possible sum = 181 + 183 + ... + 199 = 1900 So, total number of distinct values of A = (1900 - 100)/2 + 1 = 901 The number of ways in which one can select six distinct integers from the set {1, 2, 3, ...., 49} such that no two consecutive integers are selected, is (A) C(49, 6) - 5C(48, 5) (B) C(44, 6) (C) C(25, 6) (D) C(43, 6) a_b_c_d_e_f_g where a,b,c,d,e,f are the gaps and "_ " are the no's a+b+c+d+e+f+g= 43 except a,g all must be atleast 1 a+b+c+d+e+f+g= 38 ans= 44C6 Area of triangle when length of three median is given: There is a direct formula derived from apolinus right A=(4/3)square ROOT of(sm(sm-m1)(sm-m2)(sm-m3)) Where sm= (m1+m2+m3)/2 m1,m2,m3 are medians 6 gallons of wine are drawn from a cask and replaced by 6 gallons of water. 6 gallons of the mixture are next drawn and again replaced by 6 gallons of water. If the ratio of wine to water in the cask is now 81 : 19, how much wine was there in the cask initially considering there was no water in the cask at that time? (1-Q/P)^2 = 81/100 ................. 100 because, total solution = 81+19 = 100 (1-6/x) = 9/10 1. 60 gallons 2. 57 gallons 3. 36 gallons 4. 48 gallons (1-Q/P)^2 = 81/100 ................. 100 because, total solution = 81+19 = 100

(1-6/x) = 9/10 A string of 2009 digits begins with a "6". Any number formed by two consecutive digits is divisible by either 17 or 23. 1. What is the last digit in this sequence? a. 6 b. 9 c. 1 d. 4 e. None of these 2. What is the last digit if the sequence had 2010 digits? a. 2 b. 3 c. 4 d. 5 e. None of these The series is 6923469234...repeating So 2009th digit is 3 & 2010th digit is 4 15. If [x] stands for the greatest integer less than or equal to 'x' , then what is the value of : [ 1/2 + 1/1000] + [ 1/2 + 2/1000] + [ 1/2 + 3/1000] + .....+ [1/2 + 999/1000] ? Up to 0.5 value it is 0 only that means 1/2 + 16. If the roots of the equation x^3-a.x^2+ bx - c = 0 are 3 consecutive integers , then what is the smallest possible value of b ? It is -1. If p, q and r are roots are equation; then: b = pq + qr + pr Putting p as (q-1) and r as (q+1) b = (q-1)q + q(q+1) + (q-1)(q+1) b = 3q^2 - 1 When q = 0; we get min(b) = -1 For a cubic equation a*x^3 + b*x^2 + c*x + d = 0 Product of roots = -d/a Sum of roots = -b/a And sum of product of 2 roots at a time = c/a For any polynomial equation, a*x^n + b*x^(n-1) + .... + k = 0 Sum of roots = -b/a Product of roots = (-1)^n * (k/a) In an office, where working in at least one department is mandatory, 78% of the employees are in operations, 69% are in finance, 87% are in HR. What are the maximum and minimum percentages of employees that could have been working in all three departments?? There is a formula for such questions. We have 3 departments with 78%m 69% and 87%.

Now, 100 - 78 = 22 100 - 69 = 31 100 - 87 = 13 So, minimum for working in all three = 100 - (22+31+12) = 34 maximium is simple as 13 percent is required to make 100 with 87. So take this 11 from 78=78-11=67 & 2 from 69=67 minimum is to reduce overlap=78+69+87-200=34 Let I be the % of employees working in at least one department, so I = 100 II be the % of employees working in at at least two departments III be the % of employees working in all three departments I II III => I + II + III = 78 + 69 + 87 = 234 Now, to mimimize III put I and II 100 => III(min) = 234 - 200 = 34 To maximize III, put II = III (as II III) and I has to 100 => 2*III(max) = 234 - 100 => III(max) = 67 How many integers n are there between 0 and 10^99 ,such that the unit digit of n^3 is 1? 10^98. Each number from 1 to 10 results in distinct unit digit for cube. So, each digit in cube appears 1/10 times. So, from 1 to 10^99 it will appear 10^98 times. A circle circumscribes a triangle with sides 16 cm, 30 cm and 34 cm. Radius (in cms) of the circle is ??? S = 40 Area = root (40 * 24 * 10 * 6) Area = 240 Area = abc/4R R = abc / (4 * Area) R = 16 * 30 * 34 / (4 * 240 ) Radius = 17 How many values of (a,b,c) are possible such that (a+b+c -6) is a multiple of 4 where all a ,b ,c are <=6 and greater than 0? when a+b+c=6

a'+b'+c'=3, as >0 5c2=>10 ways when, a+b+c=10 a'+b'+c'=7 9C2 -> 36 when a+b+c=14 a'+b'+c'=11 let take,a'=6-a'' and likewise,where a'' can be 0 to 5 a''+b''+c''=7 9C2 -{3} when a'',b'' or c'' > 6 =33 ways when a+b+c=18 1 way. total = 1+ 36+10+ 33= 80 anuj and agraj walk up an escalator,which moves up at a constant speed . anuj climbs 5 steps for every 3 that agraj climbs. anuj gets to the top of the escalator in twenty-five steps,while agraj(his slower pace lets the escalator do a little more work)takes only 18 steps to reach the top. in how many steps would anuj reach the bottom from top,on same escalator if he doubles his speed while returning. an = 5 steps per unit time ag = 3 steps per unit time speed of escalator = a steps per unit time effectively anuj is traveling (5+a) speed and angrej is travelig at (3+a) speed 25 steps is what anuj takes -- so 25/5 is time taken 18 steps in what angrej takes -- s0 18/3 is time taken 5(5+a) = 6(3+a) 7=a so now length of escaltor is -- 60 anuj doubles his speed to 10 steps per unit time so now his effective speed is 10-7 = 3 steps per unit time time taken = 60/3 = 20 unit time

steps taken -- 20x10 = 200 The no of natural no.s of 2or more digits in which digits from left to right are in increasing order is a)127 b)128 c)502 d)501 e)512 9C2 + 9C3 + 9C 4 + .. + 9C9 = 2^9 - (9C0 + 9C1 ) = 502 when we select n numbers from 9 we can form increasing order in only one way. If x=(a-1)(b-2)(c-3)(d-4)(e-5), where a,b,c,d,e are distinct natural numbers less than 6. If x is non zero integer, then the number of sets of possible values a,b,c,d and e are 1. 76 2.119 3. 44 4. 4^5 As a,b,c,d,e can be natural and less then 6{1,2,3,4,5} but x cant be 0 Its a classical example of dearrangements. A cant take 1 B cant take 2 C cant take 3 D cant take 4 E cant take 5 So its like filling 5 letters in 5 envelopes such that no letter goes to the designated envelope. Dearrangements of 5 = 5!{1 - 1/1! + 1/2! - 1/3! + 1/4! - 1/5!} = 44. There are threee different types of wine, costing 48,55, and 75 in INR respectively. The bartender wants to make a wine that will cost Rs. 67 per pag. In what ratio should he mix all the three wines ? Options, 4:1:11 8:1:22 4:1:22 8:1:22 48x+55y+75z=67x+67y+67z 8z-12y=19x 4(2z-3y)=19x Since 19 is prime no and 4 is not its factor so one has to get 19 from 2z-3y so z=11 y=1 and x=4 Hence ratio is 4:1:11

How many ordered triplets (x, y, z) of positive integers less than 10 is the product xyz divisible by 20? (1) 72 (2) 120 (3) 90 (4) 102 (5) none of these My take is 102. We need xyz to be divisible by 20. 20 = 2^2 * 5 => We need at least one 5 and at least two 2s. Case 1: Two 5s. We will have 5, 5 and a. Here, a can be only 4 or 8. So, with 4 we have 3 ways and with 8 also we have 3 ways. Total 6 ways. Case 2: One 5 Now, with one 5 and one 4; we get 20. So, remaing digit can be filled in 8 ways (excluding 5 as we have only one 5). When other digit is not 4, we get 7*6=42 ways. When other digit is 4; we get (5, 4, 4) => 3 ways. Hence total 45. With similarly, with one 5 and one 8, we get 39 ways. (6 less ways as (5, 8, 4) is already considered) And now, with one 5 and without 4 or 8; we can use only 2 and 6. So, we get (5, 2, 6) => 6 ways (5, 2, 2) => 3 ways and (5, 6, 6) => 3 ways. So, total ways = 6+45+39+6+3+3 = 102 Second approach - 5,8,(7 choices)->7*6=42 5,4,(7 choices)->42 but 3 is common for (5,8,4)=6 so 78 now Then So we need to take two can be 2 & 6 so 6 more We have 84 Now the number can also be in the for x,x,y (two same) so 5,5,(4,8 )>6 ways (5,2,2)(5,4,4)(5,6,6)(5,8,8 )=>12 ways more so add 18 84+18=102 Two kinds of rice are mixed in the ratio 1:2 and 2:1 and then they are sold fetching the profit of 10% and 20% respectively. If they are mixed in equal ratio and the individual profit percents on them are increased 4/3 and 5/3 time respectively, then the profit % would be here mixing ratio is cost based.

Say, x% is profit on type 1 and y% on type 2. And cost price is 100 for both. Initially, we mix them in ratio as 1:2 and get 10%. So, x+2y = 3*10 = 30 And then we get 2x+y= 3*20 = 60 => x=30 and y=0. And then, we mix them 1:1 and increase profit on type 1 by 4/3. So, it will become 40%. So, we will get 40 rs from type 1 and 0 rs from type 2. So, we will earn 40 rs on 200 rs. So, profit should be 20%. The price of a pen and book increased by 23% and 17% respectively. Before the price rise, the ratio of the cost of the book to the cost of the pen was 14 : 9. If the total cost of 18 books and 28 pens before the price rise was Rs.P, what is their total cost now? Observe price ratio = 14:9 and purchase quantity ration = 18:28 = 9:14 And do the problem for 14 pens and 9 books assuming prices to be 9 and 14. So, 14*9 + 9*14 = 2*14*9 = P So, increased cost 14*9*(123/100) + 9*14*(117/100) = 14*9*(240/100) = = 2*14*9 * (120/100) = 1.2P On giving 3 pencils free with every 5 pens bought, a shopkeeper makes a profit of 20% and on giving 6 pencils free with every 2 pens bought, he suffers a loss of 25%. Find the approximate profit percent made by the shopkeeper when he gives 4 pencils free with every 6 pens bought. (Assume that the pencils are identical and the same applies to the pens.) a. 18% b. 20% c. 24% d. 16% let C.P. pencil =x c.p pen = y Cp of both 100 3x+5y =100 sp 120 that is on pen only so 5y=120 SP of pen =24 6x+2y sp is 24*2=48 so cp is 100*48/75 =64 y=17 x=5 4*5 +6*17 = 122 SP 144

profit = 22/122*100 Thirty six numbers are filled in the cells of a matrix as shown in the figure given below. Six numbers are chosen from the matrix such that no two numbers belong to the same row or the same column. In how many ways can the numbers be chosen? 123456 7 8 9 10 11 12 13 14 15 16 17 18 19 20 21 22 23 24 25 26 27 28 29 30 31 32 33 34 35 36 a. (6^2 5^2 4^2 3^2 2^2 1^2) b. (6 6 6 6 6 6) c. 720 It is not 36*25 From 1st row, you can select any number so there are 6 ways, from 2 nd row have to leave the column from which 1st number is select so 5 ways and so on So total ways are 6*5*4*3*2*1 = 720 Ashish is given Rs. 158 in one rupee denominations. He has been asked to allocate them into a number of bags such that any amount required between Re.1 and Rs. 158 can be given by handing out a certain number of bags without opening them. What is the minimum number of bags required? 1. 11 2. 12 3. 13 4. None of these For such questions, use power of 2 method. Total number of coins = 158 Each bag can have : 2^0 = 1 coin, 2^1 = 2 coins, 2^2 = 4 coins, 2^3 = 8 coins, 2^4 = 16 coins, 2^5 = 32 coins , 2^6 = 64 coins total = 127 left = 31 Number of bags = 8 Ex : Suppose we need Rs. 119. First use the 31 coins bag. 119- 31 = 88 To get 88 coins, we need 64 + 16 + 8 coins. P.S. : Can be tested for any arbitrary number between 1 to 158, say, 45, 76,83 and so on.

Therefore, total number of bags = 8 Option (4) How many subsets of the set {1, 2, 3, 4 ... 30} have the property that the sum of the elements of a subset is less than or equal to 232? Sum of all elements of set S = 465 For any subset having sum of elements x, there will be one complimentary set which is having sum of elements equal to (465 x). For Example let a set { 1,2,3,4} then subset which is having sum =3 will be {1,2} {3} (2) then subset having sum (1+2+3+4) -3 =7 will also be 2 {3,4} {1,2,4} So, number of subsets having sum of elements x = Number of subsets having sum of elements (465 - x) => No of subsets having sum 1 = No of subsets having sum 464 No of subsets having sum 2 = No of subsets having sum 463 . . No of subsets having sum 232 = No of subsets having sum 233 Adding them we will get, No of subsets having sum less than or equal to 232 = No of subsets having sum more than 233 = 2^30/2 = 2^29 Anant purchased x chocolates for Rs. y, where y is a natural number. The shopkeeper had offered to give him x+10 chocolates for Rs. 2. Anant declined the offer though it would have resulted in a saving of 80 paise per dozen chocolates for him. Which of the following can be the number of chocolates purchased by Anant? 1,3,5,15 Clearly anant purchased x chocolates for 1 Rs. Now going with options, Suppose he purchased 5 chocolates in 1 Rs. So 5+10 = 15 chocolates in 2 Rs. On 15 chocolates he saved 100 paise. => soon 12 chocolates he will save (100/15)*12 = 80 paise Therefore 5 chocolates When a no. n (n>500) is successively divided by 3,5,7, it leaves remainder 2,3,6 respectively. find the remainder when n is divided by 35? 105k+83 = 503% 35 = 13

In how many different ways can four red coloured beads and six green coloured beads be arranged in a row such that there is exactly one red coloured bead between two green coloured beads? Such an arrangement cannot be achieved.. With 6 grees coloured bead, we will have 5 gaps. So, atleast one pair of green bead will have 0 red beads between them.. So, exactly 1 condition fails. If condition is exactly 1 or 0; then my take is 7P4 * 6! _ G_ G _ G _ G _ G _ G _ There are 7 places for 4 red beads. So, they can be arranged in 7P4 ways. And 6 green bead can be arranged in 6! ways. Hence, total ways = 7P4 * 6! If the first 99 natural numbers are written side by side to form a new number 12345678 . 9899, then how many minimum numbers are to be removed so that, the new number is completely divisible by 11? considering single digit numbers first, sum at odd places= 1+3+5+7+9+9(1+2+...+9)= 25+9*45=430 sum at even places= 2+4+6+8+10(1+2+.....+9)= 30+10(45)=470 sum => even = 470 and odd => 430 odd - even = -40 (Sum of digits at odd position from right) - (Sum of digits at even positions from right) = -40 even - odd= 40 remove 99 -> diff =40 remove 98-> diff =41 remove 97-> diff =43 remove 96-> diff =46 remove 95-> diff =50 remove 94-> diff =55 we should remove 6 numbers. (99, 98, 97, 96, 95, 94) A teacher must divide 221 apples evenly among 403 students. What is the minimal number of pieces into which she must cut the apples?(whole uncut apple is 1 apple) Let each apple is cut into 'n' part. And say, each student gets 'x' pieces. So, 221*n = x*403

=> 13*17*n = x*13*31 => We can take x=17 and n=31 So, apple needs to be divided in 31 parts. How many pairs of positive integers m, n satisfy 1/m+4/n=1/12 where n is a odd integer less than 60 3 6 4 7 5 m=12n/n-48 n<60 and odd...only 49, 51, 57 satisfy the conditions Farmer John has 5 cows , 4pigs and 7 horses. How many ways can he pair up the animals so that every pair consists of different species?Assume all animals are distinguishable from each other? There will be three kinds of pair:C - H ~ a in number C - P ~ b in number H - P ~ c in number a + b = 5 (No. of pair in which cow is present) b + c = 4 (No. of pair in which pigs are there_) a + c = 7 (similarly) => a = 4, b = 1, c = 3 So, 4 cows can pe paired with 4 horses in C(5, 4)*C(7, 4)*4! = 4200 ways Now, 1 cow can be paired with 1 pig in 1*4 = 4 ways Now, 3 horses can be paired with 3 pigs in 3! ways So, answer will be 4200*4*6 = 100800 ways

A 3-digit natural number abc, where a, b and c are distinct digits, when increased by 33.33% becomes cab. When cab is increased by 33.33% it becomes bca. How many such numbers are there? Three numbers are: 100a + 10b + c 100c + 10a + b 100b + 10c + a

We can clearly see that b > c > a Now, (second number) = (first number) *(4/3) So, second number is a multiple of 4. And (third number) = (first number) * (4/3) * (4/3) So, third number is a multple of 16 and is divisible by 9. So, a+b+c has to multiple of 9. It can be 9 or 18. It can't be 27 as number are distinct. 100c + 10a + b is multiple of 4 => 10a + b is multiple of 4 => b is even 100b + 10c + a => Similarly, a is even. So, with a and b even; we have: b, a => (8,6), (8,4), (8,2), (6,4), (6,2), (4,2) Now, taking sum of a, b and c as 9 or 18 and (b > c > a); there are only two possibilities. abc => 243 or 486 How many ordered triplets (a,b,c) exist such that LCM(a,b)=1000; LCM(b,c)=2000 and LCM(c,a)=2000? 1) 32 2) 28 3) 24 4) 20 LCM(a, b) = 1000 = 2^3*5^3 LCM(b, c) = 2000 = 2^4*5^3 LCM(a, c) = 2000 = 2^4*5^3 a = 2^x1*5^y1 b = 2^x2*5^x2 c = 2^x3*5^x3 So, x3 = 4 => At least one of x1, x2 has to be 3, else LCM(a, b) will not be 1000. SO, 4^2 - 3^2 = 7 ways Now, at least two of y1, y2, y3 should be 3, else one of the LCM's will not be as desired. So, 3*4 - 2 = 10 ways So, answer will be 7*10 = 70 ways. Lets first Factorize of 1000. 1000 = 2^3 * 5^3 2000 = 2^4 * 5^3 Since LCM of A and B is 1000, they at the most contain 2 three times and 5 three times.

Also the LCM of terms containing C is 2000, the term C should contain 2 four times, as A and B can contain 2 only three times. so C should either be 2^4 = 16, 2^4 * 5^1 = 80 2^4 * 5^2 = 400 2^4 * 5^3 = 2000. This option is not possible as A and B should be within 1000(their LCM is 1000) Case1: When C = 16, Either of A or B should have a 5^3. So A or B can be 5^3 = 125 5^3 * 2^1 = 250 5^3 * 2^2 = 500 5^3 * 2^3 = 1000 Case1: When C = 80, Either of A or B should have a 5^3. So A or B can be 5^3 = 125 5^3 * 2^1 = 250 5^3 * 2^2 = 500 5^3 * 2^3 = 1000 Case1: When C = 400, Either of A or B should have a 5^3. So A or B can be 5^3 = 125 5^3 * 2^1 = 250 5^3 * 2^2 = 500 5^3 * 2^3 = 1000 Now Let one term say A be = 125, then B can only be 1000. LCM(A,B) = 1000 If A = 250, B = 1000 A = 500, B = 1000 A = 1000, B = 1000 This means that either of the terms A or b should be 1000. The other term can be one of 125, 250, 500 or 1000. C can either be 16, 80 or 400. So number of combinations = If A = 1000, 1*4*3 = 12 (A can take 1 value, B can take 4 and C can take 3) If B = 1000, 3*1*3 = 9 ( A can take 3 values (excluding 1000, as it was already included in the previous combination), B 1 and C 3) Total Number of triplets = 12 + 9 = 21 (1000, 125, 16), (1000, 250, 16), (1000, 500, 16), (1000, 1000, 16)

(1000, 125, 80), (1000, 250, 80), (1000, 500, 80), (1000, 1000, 80) (1000, 125, 400), (1000, 250, 400), (1000, 500, 400), (1000, 1000, 400) (125, 1000, 16), (250, 1000, 16), (500, 1000, 16) (125, 1000, 80), (250, 1000, 80), (500, 1000, 80) (125, 1000, 400), (250, 1000, 400), (500, 1000, 400) 1^7 + 2^7 + 3^7 + ....+100^7 is divided 202. what is the remainder ? 1^k + 2^k + 3^k...+ n^k is divisible by n(n+1)/2 when k is odd So, in the given question the whole term will be divisible by 100*101/2 => 50*101 => 25*202 Hence the term will be divisible by 202 So, remainder will be 0

A train starts from Delhi at a : b oclock (i.e. b minutes after a oclock). It reaches Chandigarh on the same day at b : c oclock after taking exactly c hours and a minutes. How many different values of a are possible? All the times are given in 24-hour clock format. 60a + b + 60c + a = 60b + c 61a = 59(b - c) So, only possibility is b = c and a = 0 So, 23 cases (b = c = 1, 2, 3, 4, ...., 23), 24 is not possible as then it will not be in the same day. set S= {1,2,3,4,5,6,7,8,9,10} then how many sunsets of S are possible that have the sum of the elements greater than 27 Sum of all elements of set S = 55 For any subset having sum of elements x, there will be one complimentary set which is having sum of elements equal to (55 - x). So, number of subsets having sum of elements x = Number of subsets having sum of elements (55 - x) => No of subsets having sum 1 = No of subsets having sum 54 No of subsets having sum 2 = No of subsets having sum 53 . . No of subsets having sum 27 = No of subsets having sum 28 Adding them we will get,

No of subsets having sum less than or equal to 27 = No of subsets having sum more than 28 = 2^10/2 = 2^9 = 512 N = (A7A)^17 is a perfect square. Which of the following statement is FALSE ? (1) A is an even digit. (2) A is divisible by 3 (3) When N is divided by 13 we get remainder 3. (4) None of these A7A has to be a perfect sq => A = 6 So, option 3 false N = 4^7 5^9 + 2^4 7 + 3 5^3 + 2^6 5^8 How many distinct digits are there in the number N? (a) 8 (b) 7 (c) 5 (d) 6 = 32*10^9 + 84 + 375 + 25*10^6 = 32025000459 So, 6 distinct digits How many arrangements of the letters of the word CATASTROPHE are there in which both the As appear before both the Ts? (a)11!/4! (b) 11!/4!(2!)/^2 (c)11!/6 (d)11!(2!)^2/4! Total possible arrangements are 11!/(2!*2!) Just consider the arrangements of AATT, among all six possibilities only one is permissible. So, 11!/{2!*2!*6} = 11!/4! a^6 + b^6 is a prime number. If a and b are distinct integers, then what is their sum? (a) 0 (b) 2 (c) 1 (d) Data Inconsistent a^6 + b^6 = (a^2 + b^2)(a^4 - a^2b^2 + b^4) = (a^2 + b^2){(a^2 - b^2)^2 + a^2b^2} Since both a and b are distinct integers, there is only one possibility when a^2 + b^2 = 2, i.e, (a, b) = (1, -1) or (-1, 1) So, sum will be 0 S1 = {2, 4, 6, 8, .................... 800} S2 = {3, 6, 9, 12, .................. 900}

If S3 = S1 S2, then what will be the 105th element of S3 if all its elements are arranged in increasing order? S1 S2 is a set comprising of all numbers which are multiples of 2 or 3 (or both) Now, we know that for a number N, there are N(1 - 1/2)(1 - 1/3) = N/3 numbers which are coprime to 2 and 3 and less than N So, there are 2N/3 numbers less than or equal to N which are multiples of 2 or 3 (or both). => 2N/3 = 106 => N = 159 is 106th such number => 158 should be the 105th number There are 545 people numbered from 1 to 545 are standing around a circle. That 1,2,3,....545,1 Now every alternate people is eliminated. Like 2,4,.. 1.Which is the number of last man standing ? 2. Say for 2nd round it was every 300th people getting eliminated. Now one proficient math teacher does some quick calculation and find that last no standing would be 437 if there were 542 people. What would be the no of last man standing for 545 people ? Options for second one:3 197 249 437 543 If we have 2^n persons, then 1st person will be the last man standing. But we have 545 persons, so we need to remove 33 persons so that we will be left with 512 persons (closest exponent of 2 to 545). So, we just have to find the person standing next to the 33rd person eliminated. Now, 2nd person is 1st eliminated, 4th person is 2nd one, ......, 66th is the 33rd one. So, person standing next to 33rd person eliminated is 67th person. So, 67th person will be the last to get eliminated.

Problem:- If a man lies 1 out of 4 times throws a dice and reports a 6. What is the probability of it actually being a six? Solution:- Let E be the event that the man reports that six occurs in the throwing of the die and let S1 be the event that six occurs and S2 be the event that six does not occur. Then P(S1) = Probability that six occurs = 1/6 P(S2) = Probability that six does not occur = 5/6 P(E|S1) = Probability that the man reports that six occurs when six has actually occurred on the die = Probability that the man speaks the truth = 3/4

P(E|S2) = Probability that the man reports that six occurs when six has not actually occurred on the die = Probability that the man does not speak the truth = 1 - 3/4 = 1/4 Thus, by Bayes' theorem, we get P(S1|E) = Probability that the report of the man that six has occurred is actually a six = {P(S1)* P(E|S1)}/ {P(S1) P(E|S1) + P(S2 ) P (E|S2)} = (1/6)*(3/4)/{(1/6)(3/4) + (5/6)(1/4)} = 3/8 Hence, the required probability is 3/8 Crime master GOGO, riding on a motorcycle, starts at the back of a 2km train as its front end enters a 4km tunnel. Both GOGO and the train travel at constant speed and GOG exits the tunnel just as the train is entirely in the tunnel. When the front end of the train emerges from the tunnel, GOg turns instantly and heads back towards the train. How many meters from the tunnel does GOGO meet the front end of the train??? When train is completely inside tunnel; it has traveled 2 kms. While Gogo has traveled 6 kms. Now, when train starts to exit tunnel (i.e. 4 kms.) Gogo should have traveled 4*3 = 12 kms. So, he is 6 kms ahead of train. Now together they need to cover 6 km. So, train will cover 1.5 and gogo will cover 4.5 kms. So, they meet 1.5 kms ahead of tunnel. if one root of the eqn (I-m)x^2+Ix+1=0 is double of the other and is real, find the greatest value of m. options: 1. 9/8 2. 8/7 3. 8/6 4. 7/5 5.5/7 If a and b are roots, then ab = 1/(I - m) = 2a^2 a + b = I/(m - I) = 3a => 2I^2/9(I - m)^2 = 1/(I - m) => 2I^2 - 9I + 9m = 0 => 81 - 72m 0 => m 9/8 Find the number of positive integral solutions of x + y + z + w = 20 under the following conditions:

(i) Zero values of x, y, z, w are include (ii) Zero values are excluded (iii) No variable may exceed 10; Zero values excluded (iv) Each variable is an odd number (v) x, y, z, w have different values (zero excluded). 1.23C3 2. a+b+c+d=16.. 19C3 4. 2a+1+2b+1+2c+1+2d+1=20 a+b+c+d=8.. 11C3 5. Best way is to count all the cases manually and then permute. Else:x + y + z + w = 20 x' + y' + z' + w' = 16 So, C(19, 3) ways When all four equal - 1 case When three equal - 5*4 cases (0 to 5 except 4) When two equal and two equal:- Say x = y = a, z = w = b a+b=8 8 cases (as a = b is already counted) So, 8*6 ways When two are equal and others are not Say, x = y = a, then 2a + z + w = 16 So, 17 + 15 + ... + 1 = 81 cases, but we need to subtract those cases also when all are equal, when z or w is equal to a and when z = w So, 81 - 1 - 8 - 8 = 64 cases So, 64*6 cases Hence answer should be C(19, 3) - 1 - 20 - 48 - 64*6 = 516 Example: (1) In how many ways can 4 red balls and 7 blue balls be arranged such that there is at most 1 red ball between 2 blue balls? (a) When balls are same: _ B _ B _ B _ B _ B _ B _ B _ => There are 8 places for red balls.

Say, they are numbered from 1 to 8. And we need a combination of 4 places. So, it is like in how many ways, can 4 numbers be selected from 8 numbers => 8C4 (b) When both types of balls are distinct: Now, not only we need to select 4 balls, but need to permute them as well. It means, say we have one combintaion of positions as (1, 2, 3, 4). As balls are different/distinct; they can be arranged in 4! ways. So, total ways for red balls will be 8C4 * 4! ... each combination gives us 4! ways. Now, it is same as 8P4. This is because, 8P4 = 8C4 * 4! And also, 7 distinct blue balls can be arranged in 7! ways.. (Like 7 people are arranged in a row) So, total ways = 7! * 8P4 (c) When red balls are distinct and blue balls are same: Now, red balls have to take 4 places from 8. And as they are distinct, we will have 8P4 ways. Blue balls are same, so only 1 way for them. So, total ways = 8P4 (d) When blue balls are distinct and red balls are same: Red balls in 8C4 ways. And blue balls in 7! ways. So, total ways = 8C4 * 7! Consider four digit numbers for which the first two digits are equal and the last two digits are also equal. How many such numbers are perfect squares? 1 2 3 4 5 0 Only 1 such numbers is 7744. If last two digits of a square are equal, they can be either 00 or 44. Here, 00 does not fit with any number. With last two digits of number as 44, number has of form (50k + 12) So, we need to check for numbers, 38, 62 and 88. No need check more as 112^2 > 4 digit numbers 88^2 = 7744 While traveling a certain distance if you travel half of the DISTANCE at 'a' kmph and the other half at 'b' kmph, you take 'x' hrs , If you travel half the TIME at 'a' kmph and for the remaining half time at 'b' kmph , you take 'y' hrs. Then : 1) x=y 2) x>=y 3) x<=y 4) NOT

Average speed in first case is s1 = d/(d/2a + d/2b) = 2ab/(a + b) Average speed in second case is s2 = (ta/2 + tb/2)/t = (a + b)/2 Now, we know that AM HM => s2 s1 => x y Funda 1: Average Speed We know that the average speed during a journey is given by (Total Distance Covered) / (Total Time Taken); but there are a few special cases which might help in solving questions, - If the distance covered is constant (d1 = d2 = d3 = dn) in each part of the journey, then the average speed is the Harmonic Mean of the values. SpeedAvg = n / (1/s1 + 1/s2 + 1/s3 + 1/sn) - If the time taken is constant (t1 = t2 = t3 = tn) in each part of the journey then the average speed is Arithmetic Mean of the values. SpeedAvg = (s1 + s2 + s3 +sn)/n Funda 2: Using Progressions (Arithmetic & Harmonic) In many questions, you will come across a situation when a person is going from point A to point B at various speeds and taking various times. We know that if distance is constant, speed and time are inversely proportional to each other. But this information can also be used to deduce the following two facts, - If the various speeds which are mentioned are in AP, then the corresponding times taken will be in HP. - If the various speeds which are mentioned are in HP, then the corresponding times taken will be in AP. Let us use these ideas to solve couple of quant questions. Example 1 Arun, Barun and Kiranmala start from the same place and travel in the same direction at speeds of 30, 40 and 60 km per hour respectively. Barun starts two hours after Arun. If Barun and Kiranmala overtake Arun at the same instant, how many hours after Arun did Kiranmala start? [from CAT 2006] (Some useless information: Arun Barun Kiranmala is a 1968 Bangladeshi film. Now you can guess what inspires the CAT question setters. Here is a song from the film.)

Solution As you can see that the speeds are in HP, so we can say that the times taken will be in AP. Time difference between Arun and Barun is 2 hours, so the time difference between Barun and Kiranbala will also be 2 hours. Hence, Kiranbala started 4 hours after Arun. Example 2 Rishi Kapoor can swim a certain course against the river flow in 84 minutes; he can swim the same course with the river flow in 9 minutes less than he can swim in still water. How long would he take to swim the course with the river flow? Solution Let us say Speed of Rishi Kapoor in still water is RK and Speed of the river is R. Hence, Rishi Kapoors speeds against the river flow, in still water and with the river flow are, RK R, RK and RK + R. As you can see, they are in AP. Hence, the corresponding times taken will be in HP. Let us say that the time taken to row down with the stream is t, then 84, t+9 and t are in HP. So, t + 9 = (2 * 84 * t) / (84 + t) t2 + 93t + 756 = 168t t2 75t + 756 = 0 t = 63 or 12

Funda 3: Special Case Let us say that two bodies a & b start at the same time from two points P & Q towards each other and meet at a point R in between. After meeting at R, a takes ta time to reach its destination (Q) and b takes tb time to reach its destination (P). Then, Sa / Sb = (tb / ta) Also, the time taken by a & b to meet (i.e. to reach point R from P & Q respectively) is given by, t = (ta * tb) Note: The same formulae will be valid if two bodies a & b start at different times from two points P & Q towards each other. They meet at a point R in between after travelling for ta and tb time respectively. After meeting, they take the same amount of time (t) to reach their respective destinations (Q & P).

[(2^2)+1]*[(2^4)+1]*[(2^ 8 ) +1]*...................*[(2^1024)+1] (a)1024 (b)2048 (c)2^2048 (d)(2^2048 )+1 (e)(2^1024)-1 Now my take is (2^1024)-1 (2^2+1)*(2^4+1)*....*(2^10+1) => All terms are odd => Product is odd Only 2 options are possible: (2^2048 )+1 and (2^1024)-1 Now, 2^2 + 1 = 5 So, number is divisible by 5. Let's check which option is: 2^4 = 16 leaves remainder by 5 as 1. So, 2^(4k) will leave remainder by 5 as 1. So, (2^2048 )+1 will leave remainder by 5 as 2. And (2^1024)-1 will be divisible by 5. And hence (2^1024)-1 should be the answer. abcd is a rectangle with sides 384 and 240. Squares of area equal to unity are formed by maing a grid inside the rectangle. Now a line is drawn connecting the 2 oppositite verticles ( diagonal). how many squares does this line pass thru ? 575 623 624 576 Its like, suppose we have a rectangle of dimension a*b, where a and b are co-prime, and it is divided into a*b unit squares by drawing lines parallel to the edges of rectangle. That means we have total a + 1 parallel lines in one direction and b + 1 parallel lines in other Now, we need to find the number of squares that the diagonal of the rectangle will intersect. If we start from one corner, then whenever the line intersects an already existing lines (among a + 1 and b + 1 parallel lines), then we are entering a new square. So, we just have to count the number of times when this line (diagonal) intersects already existing lines. But there might be a possibility that the diagonal intersects two line simultaneously, i.e, at the intersection point of two existing lines. But since a and b are co-prime. its possible only at extreme corners. Now, if we start moving from one corner, then the line will intersect a lines in one dimension and b in other. But we need to subtract one as at the extreme of the line or at the vertex of the rectangle it is

intersecting two lines simultaneously. So, (a + b - 1) squares will be intersected Now, if a and b are not coprime, then Suppose a = kx and b = ky, where k is GCD, then we can divide it in k number of x*y rectangles along the diagonal So, answer will be (x + y - 1) for one So, for k it will be k(x + y -1) = kx + ky - k = a + b - GCD(a, b) how to find the no. of integer co-ordinates within a given area . for eg; say x^2+y^2 = 25 Nachiket approach:We just need to check for 1 quadrant, then multiply by 4 and add origin. Better First quadrant: x = 5 => y = 0 x = 4 => y = 3, 2, 1 x = 3 => y = 4, 3, 2, 1 x = 2 => y = 4, 3, 2, 1 x = 1 => y = 4, 3, 2, 1 => Total 16 So, Total in all quadrants = 16*4 + 1 (for origin) = 65 can t multiply values of first quardant by 4 as points on line are also common Neetu :X=4 Y=1 x=3 y =1,2 x=2 y =1,2,3 x=1 y =1234 so total 10*4+17 (on axis) = 57

Six people have to go to attend an event in three different vehicles each of which can accommodate a maximum of six persons, In how many ways can they go so that they use all the three vehicles. 1,2,3 = 6C3*3C2*3! = 360 1,1,4 = 6C4*2C1*3!/2 = 90 2,2,2 = 6C2*4C2 = 90 so,total ways = 540 1 + 1/2 + 1/3 + ... + 1/2010 = m/n, where m and n are co-prime, then m is divisible by :a) 2009 b) 2010 c) 2011 d) 1001 e) None 1 + 1/2 = 3/2 .. 3 is divisible by 3 1 + 1/2 + 1/3 +1/4 = 25/12 .. 25 is divisible by 5 so in general for an even number , numerator is divisible by n + 1 so we get 2010 + 1 = 2011 on a chess board if two squares are chosen what is probability that they have a side in common?? Maximum possible cases = 64C2 Favourable cases: Now, we need to find sqaures with common sides. So, basically we need to find number of sides which are common to 2 squares. We will have 7*8*2 such sides [All the small 1 unit sides except border sides of 8*8 sqaure can be common sides. Now, we will 7 verical lines with each of them having 8 small 1 unit lines which can be shared. So, 7*8. And similarly 7*8 horizontal sides] So, P = 7*8*2/(64*63/2) = 1/18 Which of the following is /are true?? a)If [log1]+[log2]+[log3]+[log4]+......+[logN]=N,Where [x] represents the greatest integer then or equal to x,then N=108(taking log to the base 10) b)If [log1]+[log2]+[log3]+[log4]+......+[logN]=N,Where [x] represents the least integer more than or equal to x,then N=11(taking log to the base 10) c)If [log1]+[log2]+[log3]+......[logN]=N,Where [x] represents the greatest integer then or equal to x, then N=10098.(taking log to the base 100 here)

a)only a b)only b c)only c d)a and b e)a,b,c When [a] is greatest integer less than or equal to a and base is 10. [log n] = 0, for n = 1 to 9 = 1 for n = 10 to 99 = 2 for n = 100 to 999 For n = 108, it comes out to be 90 + 9*2 = 108 (so option (a) is okay) When base is 100, then [log n] = 0, for n = 1 to 99 = 1 for n = 100 to 9999 = 2 for n = 10000 to 999999 For n = 10098, it comes out to be 9900 + 99*2 = 10098 (so option (c) is okay) When [a] is least integer greater than a and base is 10, then [log n] = 1, for n = 1 to 10 = 2 for n = 11 to 99 => For n = 11, sum will be 10 + 2 = 12 (so option (b) is incorrect) EDIT;- [log 1] in last case will be 0, so option (b) is also correct. Number of numbers less than and prime to a given number: If N is a natural number such that N= a^p*b^q*c^r, where a, b, c are different prime nos. and p,q, r are different positive integers then, the number of positive integers less than and prime to N = N(1-1/a)(1-1/b) (1-1/c) e.g: how many first 1200 nos. are not divisible by 2,3 and 5?? 1200= 2^4*3^1*5^2 Therefore number of numbers not divisible by 2, 3 and 5 will be equal to the numbers prime to 1200 =1200(1-1/2)(1-1/3)(1-1/5) =1200*1/2*2/3*4/5 =320 ans Thus there are 320 such nos which are not divisible by 2, 3 and 5 Rectangles Cut By a Diagonal Line The general rule: If the lengths of sides (a x b) of the rectangle are mutually prime, the number of squares cut is a+b-1 Thus, your example: (3 x 5) gives 3+5-1 = 7 Other examples: (8 x 5) gives 8+5-1 = 12 (9 x 4) gives 9+4-1 = 12 (9 x 5) gives 9+5-1 = 13 BUT (9 x 6) DOES NOT give 9+6-1 = 14. Instead you must proceed as

follows: First divide (9 x 6) through by common factor 3 to get (3 x 2) Then apply the rule to (3 x 2) to give 3+2-1 = 4 Now multiply by the factor 3 again to get 12 (which is correct). Let's do a square figure, say (5 x 5). We divide through by 5. This gives (1 x 1). Applying the rule gives 1+1-1 = 1. Now multiply up by the factor 5 again to get 5. We know this is correct because in ANY square figure the number of squares that are cut will be equal to the side of the square. The general procedure for a rectangle (a x b) is as follows: If a and b are relatively prime the answer is a+b-1 If a and b have a common factor c, first divide through by c to get (a/c x b/c). Then apply the rule to get a/c + b/c - 1. Finally, multiply through again by c to get (a+b-c). For example, with (9 x 6) the correct answer is 9+6-3 = 12. Pigeon hole theorem if n+1 pigeons fly to n holes, there must be a pigeonhole containing at least two pigeons Well this theorem, look apparently simple and trivial, but its extremely powerful. Lets take a test of it. Example= Let A be any set of nineteen integers chosen from the arithmetic progression 1,4, . . . ,100. Prove that there must be two distinct integers in A whose sum is 104. Now how do we go about this? remember n and n+1. The hint is to make n+1=19. Something clicked? see we have 34 numbers of the form 3k+1, from 1 to 100. If we do not want a sum of 104 , we will break them in the sets of 2 integers whose sum is 104 {4,100},{7,97}..{49,55} and {1}, {52}. Clearly we have 16 two element sets and 2 one element set. So if we make a set of 19 integers, we will have to pick both the integers from atleast one of the two element sets, which will give us a sum of 104. We are done here. If you still have doubts, let me explain again, suppose you are four friends ( boys) and there are three girls. And each one of you like a girl out of the three. So at least one of the girls will be liked by two boys.

Lets solved a more involved example, wherein we need not prove a thing, but find a thing. Some people may be feeling cat does not want us to prove but find. Here is how we do that.

Example = Let there be n balls with Ram. he decides to colour one ball with colour 1, two balls with colour 2 and so on upto, fifty balls with colour 50. At the end of it , all n balls are used, and no ball is coloured twice. Ram then draws balls from the lot at random, without replacement. What is is the minimum number of balls that he must draw in order to gurantee drawing 10 balls of the same color? What the hell is his problem. Why coloring and then taking out. Stupid chap. Let us help him with the math now. see if he picks all the balls with colors which are less than 10 it will come upto (1+2+3..+9)=45. Now for the worst case he will pick 9 balls each from rest of the balls, which is 41*9 so total is 41*9+45=41*10+4=414. ( avoid multiplying, be watchful) now if he picks one more ball, atleast one of the set will be of 10. so we are done he needs to draw 414+1=415 balls. Practice Problem 1 = A circular table has exactly 60 chairs around it. There are N people seated at this table in such a way that the next person to be seated must sit next to someone. What is the smallest value of N? Practice Problem 2 = We call a set "Sum-free" if no two elements of the set add upto a third element of the set. What is the maximum size of the "sum-free" subset of {1,2,3...2n-1}? Rules:1)Every even number > 2 can be expressed as the sum of 2 primes. 4=2+2, 6=3+3, 8=3+5, 10=5+5, 12=5+7, .. , 100=3+97 2) All these are prime numbers... 31 331 3331 33331 333331 3333331 333333331 next number 333333331 is not prime 3) if a cyclic quadrilateral is given with sides of length a,b,c,d and diagonals of length e,f, then ac+bd=ef( this is known as Ptolemy's Theorem) Since MAX(x1, x2) = x, we can say that at-least one of them should be x. So, we can find the number of ways in 2 different manner:I) If x1 is x, then x2 can take (x + 1) values, i.e., 0 to x. Similarly when x2 is x, x1 can take (x + 1) values. So, total 2x + 2 ways, but we need to remove the case when both are x which is counted twice. So, 2x + 1cases

Euler's Totient Eulers totient, (n), of a a number n is the number of positive integers less than or equal to n that are coprime to n. If n = (p1)^a*(p2)^b*(p3)^c*......, where p1, p2, p3, .... are prime numbers then (n) = n(1 - 1/p1)(1 - 1/p2)(1 - 1/p3)..... For ex:147 = 3*7^2 (147) = 147(1 - 1/3)(1 - 1/7) = 84 Eulers's Theorem If {(a^(n)) - 1} is divisible by n, where (n) is Euler's totient of n and n is coprime to a. That means a^(n) = 1 (mod n) For ex:We have to find the remainder when 5^180 is divided by 209. Now, 209 = 11*19 => (209) = 209*(1 - 1/11)(1 - 1/19) = 180 => Since 5 and 209 are coprime, we can say that 5^180 = 1 (mod 209) Fermat's Little Theorem It is an extension of Euler's Theorem. If in Euler's theorem n is a prime number, then a^(n - 1) - 1 will be divisible by n is a and n are coprime. We can see that (n), where n is a prime no will be:(n) = n(1 - 1/n) = n - 1 That why when n is prime we can write a^(n - 1) - 1 will be divisible by n. An application of Euler's Theorem Suppose we have number N having digit 'a' written 'k' times, where k is Euler's totient of 'n'. We have to find the remainder when N is divided by 'n' and it is also given that n and 10 are coprime N = aaa...a (k times) = (a/9)(10^k - 1) Now, since 10 and n are coprime, we can say that (10^k - 1) is divisible by n as k is Euler's totient of 'n'. Now since n and 9 are also coprime, we can say that (a/9)(10^k - 1) will be divisible by n. That means any digit written (n) times will be divisible by n if n is coprime to 9 and 10

For ex:Find the remainder when 99999....9(54 times) is divided by 19. Now, we know that (19) = 18 and 54 = 3*18 and also 19 is coprime to 10 and 9. Hence we can say that:99999....9(54 times) is divisible by 19

Angle Bisector Theorem A classic example from one of the simcat's which makes use of both external and internal angle bisector theorems. Find DE : DB, if BC = 3 units, EB = 1 unit and DE is an angle bisector of CDB and DB is an angle bisector of ADE.

We are asked to find DE/DB, From the above figure we see that DE is internal angle bisector of Triangle(DCB) whereas DB is the External angle bisector of Triangle(DCE). So, From an external angle bisector in Triangle(DCE) we have, DC/DE = BC/BE..............i From an internal angle bisector in Triangle(DCB) we have, DC/DB = EC/EB .............ii From i & ii we get, DE/DB = 2/3.

AP / PM = (b + c) / a where, P is the incentre and a, b, c are the respective sides of the triangle. For any set of n positive numbers , Arithmetic Mean >= Geometric Mean >=Harmonic mean The equality occurs only when all the numbers are equal . This can be used if the sum(or product ) of some numbers are given and the maximum(or minimum) value of the product(or sum ) is asked ... Ex : xy=27

minimum value of 3x+4y ? taking 3x and 4y to be two numbers and applying our rule A.M : (3x+4y)/2 G.M : (12xy)^1/2=(4*3*27)^1/2 = 18 A.M >= GM 3x+4y >= 2*18 Hence the minimum value of (3x+4y) is 36 concept 3. total number of squares which can be made from size in N*N size square. = 1^2 + 2^2 + 3^3 .....N^2 like for 2*2 square we can have total 5 squares, 4 square of 1*1, and the 2*2 square itself. concept 4. total number of rectangles which can be made from N*N square. = 1^3+2^3+3^3.....n^3 concept 5. A plane( restricted) is to be divided in N distinct parts, find the minimum number of lines to do so. formula = > sigma X = N-1 X is the minimum number of lines. suppose we want to divide plane in 16 distinct parts sigma5 = 15 so answer is 5 lines. Suppose we have to find the number of ways in which an ordered pair (a, b), where a and b are natural numbers, can be choosen such that LCM of a and b is (p^x)*(q^y)*(r^z)*... Let a = (p^x1)*(q^y2)*(r^z3)*... and b = (p^x2)*(q^y2)*(r^z2)*... Since LCM of a and b is (p^x)*(q^y)*(r^z)*..., we can say that max(x1, x2) = x => one of x1 and x2 has to be x, this can be done in (x + 1)^2 - x^2 = (2x + 1) ways Similarly, max(y1, y2) = y => (2y + 1) ways & max(z1, z2) = z => (2z + 1) ways So, total number of such ordered pairs = (2x + 1)(2y + 1)(2z + 1).. Had the question been that in how many ways two numbers can be choosen such that their LCM is (p^x)*(q^y)*(r^z)*..., then the answer would be {(2x + 1)(2y + 1)(2z + 1).... + 1}/2 Number Systems (Concept 1)

Lets brush up some painted cube funda We assume the cube is divided into n^3 small cubes. no. of small cubes with ONLY 3 sides painted : 8( all the corner cubes ) no. of small cubes with ONLY 2 sides painted : A cube is painted on 2 sides means, it is on the edge of the bigger cube ,and we have 12 edges, each having n cubes. but since the corner cubes are painted on 3 sides, we need to neglect them. so in effect, for each side we will have (n-2) small cubes with only 2 sides painted. thus, then number is, 12 * (n-2) no of small cubes with ONLY 1 side painted : for each face of the cube ( 6 faces ) we have (n-2)^2 small cubes with only one side painted. and we have 6 faces in total. so th number is, 6*(n-2)^2 no of small cubes with NO sides painted : if we remove the top layer of small cubes from the big cube we will end up a chunk of small cubes with no sides painted. this number will be equal to, (n-2)^3. Also, remember for Cuboids with all different sizes, the following are the results: a x b x c (All lengths different) Three faces - 8 (all the corner small cubes of the cuboid) Two faces - There are two (a-2) units of small cubes on one face of the cuboid and there is a pair of such faces. Hence, number of such small cubes corresponding dimension a of the cuboid = 4(a-2). Similarly, for others. So, total with two faces painted = 4(a - 2) + 4(b - 2) + 4(c - 2) One face - Since each face of the cuboid is a combination two different dimensions, hence for the face which is a combination of a and b dimensions, the number of small cubes is 2* (a-2)(b-2) [multipliesd by 2 because there are 2 such faces for the combination] Similarly, for others. So, total with one face painted = 2(a - 2)(b - 2) + 2(a - 2)(c - 2) + 2(b - 2)(c - 2) Zero faces - The entire volume of small cubes except for two cubes in each of the rows and columns will not be painted at all. hence this is the simplest ... (a - 2)(b - 2)(c - 2)

You can put different integer values for number of small cubes producing different edge lengths of cuboid to get varied results. To verify for a cube, put a=b=c=L, you get Three faces - 8 Two faces - 12(L - 2) One face - 6(L - 2)^2 Zero faces - (L - 2)^3 Problems on Intersection of Straight lines , Circles, Formation of Points and Formation of Triangles, Quadrilaterals Etc Basic Concept Fundas 1. If there are n number of straight lines , They intersect each other in nc2 ways 2. If there are m number of circles , They intersect each other in 2*(mc2) ways = m (m-1)= 2p2 ways 3. When n straight lines and m circles intersect each other , they intersect in at most 2 * m * n = 2* ( no. of circles ) * ( no.of straight lines) 4. When n parallel lines intersect m straight lines , Then no. of parallelograms possible = nc2 * mc2= mn (m-1) (n-1)/4 5. There is one case when collinear and non- collinear points are given , and asked how many triangles it can formed=> The funda for this - ( Triangles that can be formed with all points ) ( Triangles formed with collinear points ) And the same funda is applied whenever such variations in condition occurs 6. For quadrilaterals standard approach is followed 1. 4 points out of 8 points are collinear . Number of different quadrilaterals that can be formed by joining this is 1. 56 2. 53 3. 76 4. 60 => total points = 8 => Collinear points = 4 , Non collinear points = 4 => Standard approach = => 4c0 * 4c4 + 4c1* 4c3 + 4c2 * 4c2

=> 1+ 16+36 => 53 Nof quads with 0 points : 1 No of quads with 1 point 4 * 4C3 = 16 No of quads with 2 points 4C2 * 4C2 = 36 so in all 53. 2. The number of points of intersection of 8 different circles is 1. 16 2. 24 3. 28 4. 56 => Guess easy now ; Standard Formula = 2* nc2= n*(n-1) = np2 => 8 * (8-1)= 8*7 => 56 3. The max . number of points of intersections of 8 straight lines 1. 8 2.16 3. 28 4. 56 => Standard formula = nc2 => 8c2 = 8*7/ 2= 28 4. The max. no of points into which 4 circles and 4 straight lines intersect is , 1.26 2. 50 3. 56 4.72 => Since max. no of points is asked => ( no. of points possible due to intersection of 4 circles with each other ) + ( no. of points possible due to intersection of 4 straight lines ) + ( no. of points possible due to intersection of 4 circles with 4 straight lines ) => 4p2+ 4c2+ 2*4*4 => 4*3+2*3+32 => 12+6+32

=> 50 5. If 5 straight lines are intersected by 4 straight lines , The number of parallelograms possible => Standard formula => 5c2 * 4c2 =>60 6. Now problems on Formation of triangles If there are 7 points out of 12 lie on the same straight line , then number of triangles thus formed is. => Here total points= 12 => These 12 points can formed 12c3 triangles => and 7 collinear points can also form 7c3 triangles => Thus total triangles possible = 12c3-7c3 => 185 There are 6 straight lines in a plane, no 2 of which are parallel and no 3 of which pass through the same point. If their points of intersection are joined, then the number of additional lines thus introduced is (a) 45 (b) 78 (c) 105 (d) none of the foregoing => solution for this => Total points formed from the intersection : (6c2)c2 = 105 Number of existing lines from these points : 6*5c2 = 60 So, 105-60 = 45 9.which one is the list containing the number of points at which a circle can intersect a triangle ??? 2, 4 2, 4, 6

1, 2, 3 1, 2, 3, 4 1, 2, 3, 4, 5, 6 => 1,2,3,4,5,6 10.if there are 7 pts. on a circle and 10 non collinear pts outside circle in same plane how many circles can be made? => Total Points = 7+10 = 17 => For a circle formation 3 points are required - 17c3+ 1( This one is added cause 7 points can give one circle ) => 7 points are collinear , so circles are omitted - 7c3 => Thus the answer is 17c3 + 1 -7c3 3 points are required for a circle. And there are 17 points. hence 17C3 circles can be formed. But 7 points are on the same circle. hence 17C3 - 7C3 +1 (+1 because 7 points are on the same circle). 11.if there are collinear 7 pts and 8 collinear pts. on other line parallel to it and 3 non collinear pts outside two lines and all are in same plane , find no. of circ can be made => Total no. of points = 7+8+3= 18 Points => Number of circles that can be made with these points = 18c3 => And since 7 points and 8 points are collinear , the circles made with it are omitted 7c3 and 8c3 => Thus answer is 18c3-7c3-8c3 A cuboid with dimensions l, b and h is painted on surface and then cut into cubes of 1cm3 sizes. Now how many cubes have none of the faces painted, how many cubes have one face painted, how many cubes have two faces painted and how many cubes have three faces painted. Ans: No of cubes with no face(side) painted is (l-2)(b-2)(h-2) No of cubes with one face(side) painted is 2(l-2)(b-2) + 2(b-2)(h-2) + 2(l-2)(h-2) No of cubes with two faces(sides) painted is 4(l-2) + 4(b-2) + 4(h-2) No of cubes with three faces(sides) painted is 8 (always constant) No of cubes with four or more faces (sides) painted is zero. If problem statement says its cube instead of cubiod with k cm sides. Then the answers will be (k-2)3, 6(k-2)2, 12(k-2), 8 and zero respectively. (Multiplication Principle) If there are n choices for the first step of a two step process and m choices for the

second step, the number of ways of doing the two step process is nm. The number of arrangements of n objects is n! The number of arrangements of r out of n objects is nPr = n!/(n-r)! The number of arrangements of n objects in a circle is (n-1)! The number of arrangements of n objects on a key ring is (n-1)!/2 The number of arrangements of n objects with r1 of type 1, r2 of type 2, ..., ri of type i is n!/(r1!r2!...ri!) The number of ways of choosing n out of r objects is nCr = n!/((n-r)! r!) The number of distributions of n distinct objects in k distinct boxes is kn. The number of ways of distributing n identical objects in k distinct boxes is (n+k-1)Cn. 1.if f(x) = ax^2 + bx + c, how to find maximum, minimum . USE first derivation test. f'(x)= 2aX+b. find value of X for which , 2ax+b= 0. when a>0, at this value of X, f(x) is minimum. when a<0 at this value of X , F(x) is maximum. now example.... f(x) = X^2 + 4X + 3. if 2X+4 = 0 X= -2 here a>0, so at x=-2, f(x) will attain minimum value. f(-2) = 4-8+3 = -1. if f(x) = -X^2 + 4X + 3 f"(x) = -2x + 4 X= 2. f(2) = -4+8+3 = 7 is the maximum value of f(x). generalizing for f(x) = ax^2 + bx + c, x=-b/2a (2ax+c=o)gives maximum or minimum value of f(x) depending upon a>0 or a<0. for more examples and explanation with graphs visit the linkFirst derivative Test second derivative test 2. f(x) = l x-a l + l x-b l + l x-c l for such questions, either at x=a or x= b or x= c or x=avg(a,b,c) will f(x) minimum. f(x) = l x-2 l + l x-8 l + l x-11 l here x= 2+8+11/3 = 7 will give the minimum value ( f(7) = 11) f(x) = l x-2 l + l x-5 l + l x-11 l here x= 5 will give the minimum value. ( f(5) = 9) 3. when sum of any quantities is constant, there product is maximum when they are equal. example. if 3x+5y=15. find maximum value of x^2*y^3. here 3x+5y=15 => 3x/2 + 3x/2 + 5y/3 + 5y/3 + 5y/3 = 15.--------------1 as I said, when sum of any quantities is constant, there product is maximum when they are equal.

here sum is constant. so when 3x/2 = 5y/3. we get maximum value of x^2*y^3. taking 3x/2 = 5y/3 putting it in 1, => 5(3x/2) = 15. =>x=2. and y = 9/5. answer is 2^2*(9/5)^3. generalizing it, how to find maximum value of x^m*y^n where ax+by=P. a,b,x,y>0 x^m*y^n is maximum when ax/m = by/ n = p/m+n 4. when the product of any quantity is constant, sum of the all the quantity is minimum, when they are equal. xy^3 = 64. find minimum value of x+12y. we need to adjust x+12y, accordingly. x+12y = x+ (12y/3)*3 now, x*(12y/3)^3= 64 *64 ( coz xy^3 = 64)-----------1 the product is constant. so the sum of the quantities will be minimum when quantities are equal. take x= 12y/3 putting it in 1, we get x= 8 =>12y/3 = 8, y = 2. minimum value of x+12y = 8+24 = 32. generalizing it, how to find minimum value of ax+by where x^m*y^n=P a,b,x,y>0 ax+by is minimum when ax/m = by/n concept of CRT : problem: find the smallest number when divided by 5 leaves 3 and when divided by 7 leaves 4 common approach divisor of 5 + 3 - 3,8,13,18,23,28,33 divisor of 7 + 4 - 4,11,18,25,32 18 is common to both series .. so we have the answer... same is the case with chinese remainder theorem find the remainder of 3^1001 divided by 1001 ...

1001 - 7*11*13 so find the remainder when 3^1001 divided by 1001 3^1001 / 7 ----> 3^5/7 , remainder - 5 3^1001/11 ----> 3/7 ,remainder - 3 3^1001/13 ----> 3^5/13 , remainder - 9 so we get 7a + 5 = 11b + 3 = 13c+9 now what is word interpretation of the above statement .. find the smallest number which when divided by 7 gives remainder 5 , when divided by 11 leaves remainder 3 and when divided by 13 leaves remainder 9? first take any two condition 11b + 3 = 13c + 9 divisor of 13 + 9 = 9,22,35,48,61,74,87,100,113,126,139 divisor of 11 + 3 - 3,14,25,36,47,58,69,80,91,102,113 so smallest number is 113 whats the next number then ? its of form LCM(11,13) + 113 = 143k + 113 so we have combined two conditions so now our job is to compare this with third one 143k + 113 = 7a + 5 143k + 108 = 7a 140K+ 3k + 105 + 3 = 7a so 3k + 3 should give 0 remainder when divided by 7

so k = 6 final remainder is hence 143(6) + 113 = 971 this is all about chinese remainder theorem to sum up , use this theorem only when denominator is factorisable to prime factors. Bases 1. A number is base N is divisible by N-1, when the sum of digits in base N is divisible by N-1 2. When digits of a number N1 in base N are rearranged to form a number N2, then N2-N1 is always divisible by N-1. 3. If a number in base N has even number of digits and that number is a palindrome, then the number is divisible by N+1 Problems

1. A number 2342a121 is in base 8 and it is divisible by 7. Find the value of a. 2. A palindromic number in base 16 will always be divisible by which number? 3. A five digit number is in base 19. It is rearranged to form another 5 digit number. The difference of these numbers will be divisible by ??

SHORTCUT METHOD TO FIND RANK OF A GIVEN WORD This shortcut method is used when the lettors of the given word are not repeated. Given word is MASTER The letters of the are M,A,S,T,E,R. Write the alphabetical order of the letters of the given word ' MASTER ' as A,E,M,R,S,T Now strike off the first letter M. A,E,M,R,S,T. Then count the no.of letters before M, and it is equal to 2,which is the coefficient of 5!. Again strike off the first letter A. A,E,M,R,S,T Then count the no.of letters before A and it is equal to 0 which is coefficient of 4! Again strike off the first letter S. A,E,M,R,S,T Then count the no.of letters before S and it is equal to 2 which is coeffcient of 3! Again strike off the first letter T. A,E,M,R, S, T Then count the no.of letters before T and it is equal to 2 which is coeffcient of 2! Again strike off the first letter E. A,E, M,R, S,T

Then count the no.of letters before E and it is equal to 0 which is coeffcient of 1! Finally add 1 to the above values to get the rank of the word MASTER as follows: 2(5!) + 0(4!) +2(3!)+2(2!)+0(1!)+1=257 The concept of rank goes like this... Say for the number of letters before S..we have A,E,M & R before it...but we have already counted A & M..so we will go with E & R...i.e 2 letters remain... Similarly in case of T..we have A,E,M,R,S before it..but we have already counted A,M,S...so only E,R are left..i.e 2 letters.. Also in case of E we have A,M,R,S,T..but we have already taken care of all of them..no letters are pending..so 0 letters.. Hope this clears your confusion..Please PM me if further clarification is required.. Basic Formulae for Sequences and Series Some more formulae1. Greatest possible sum of the A.P - It is possible only when all the terms of A.P are non- negative Tn >or equal 0 2.Least possible sum of the A.P - it is possible only when all the terms of A.P are non- positive Tn < equal 0 NO. OF SQUARES AND RECTANGLES IN A CHESSBOARD in a chessboard,there are 8*8 squares. in a 2*2 chessboard, there are 5 squares (4 small aquares, 1 big square). similarly in an n*n chessboard, there are 1^2 +2^2+....+n^2 squares. so in a 8*8 chessboard, n=8 => no.of squares = 1^2 +2^2+3^2+.....+8^2 = [n (n+1) (2n+1)]/6 (summation formula) = 204 rectangles : in a 2*2 chessboard, there are 9 rectangles (4 1*1s,1 2*2,2 2*1s, 2 1*2s) for an n*n chessboard, there are 1^3 +2^3+3^3+.....+n^3 rectangles. so for an 8*8 chessboard, there are, rectangles = 1^3+2^3+....+8^3 => no. of rectangles = [{n^2}{(n+1)^2}/4 n=8, we get no. of rectangles = 1296

**No. of rectangles that are not squares in an 8*8 chessboard => 1296- 204 = 1092 CIRCULAR MOTION Consider a circle with circumference 200 sq metres and two people A and B moving in clockwise direction with speeds 5m/s and 9m/s a) when do they meet at the starting point for the first time ? b) after how much time will they meet for the first time ? c) at how many distinct points they meet ? d) if they move in opposite direction then in how many distinct points do they meet ? ans a) time taken by A to reach the starting point for the first time is 200/5 = 40 sec, for the second time 80 seconds for the third time 120 seconds and so on ....... similarly time taken by B to reach the starting place is 200/9 for the second time 2 * 200/9 for the third time 3 * 200/9 so they meet for first time at the starting point at the LCM of their time periods which are 40 and 200/9 in this case and hence they will meet at t= 200 for the first time . ans b )this should be solved using the relative speed concept time taken for them to meet for the first time will be relative distance/ relative speed here relative distance is 200 and relative speed is 9-5=4. so ans is 200/4= 50 ans c)when 2 bodies are moving in circular motion in same direction the number of distinct points where they meet is the difference of the speeds . here it is 9-5 = 4 distinct points. the lcm of the speeds must be 1 ans d ) when 2 bodies move in circular motion in opposite direction then the number of distict points they meet is the sum of the speeds here it will be 14 . the lcm of the speeds must be 1 e) find the number of distict points at which 2 bodies with speed 4 and 8 meet when they move in clockwise direcion and when they move in anticlock wise direction ? here we first need to divide the HCF so we get 1:2 now using the formula stated above the number of distict point they meet when moving in same direction will be 2-1 = the number of distict point they meet when moving in opposite direction is 2+1 = 3 f)consider three bodies a,b,c with speeds 5,9,13 respectively moving in clock-wise direction now number of distinct points at which all three meet ?

a-b = 4 b-c = 4 a-c = 8 so number of distinct points will be the hcf of the differences which is 4 ans. g)consider three bodies a,b with speeds 5,9 respectively moving in clock-wise direction and c with speed 13 in anti-clockwise direction now number of distinct points at which all three meet ? a-b=4 a+c=18 b+c=22 hcf is 2 so they all will meet at 2 distinct points last 2 digits of a number ] divide the number by 100 then it will give the last 2 digits if division is cumbersome u may use one of the following methods: points to remember : if last 2 digits are 25 then (abcdef25)^z where z is a natural number will always give the last 2 digit as 25. if last 2 digits are 76 then (abcdef76)^z where z is a natural number will always give the last 2 digit as 76. if the last digit is 1 say the number is (abcdefg1)^thgfds then the units digit will be 1 and the tens digit will be g*s e.g (1231)^4563 last 2 digits will be 91. (sdfdsf24)^ odd = last 2 digits 24 (dfdsfd24)^even = last 2 digits 76 (dbfh26)^odd = last 2 digits 26 (dfdsaf26)^even = last 2 digits 76 few examples : last 2 digits for (71)^45 = 51

Difference of Squares : The concept is if there is any N = a^2 - b^2 and a and b are integers. So many ways a and b are chosen? If N = a^2 - b^2 = (a+b) (a-b), if you want to find integers a and b, then (a+b) and (a-b) should be both either odd or even For example if we take N = 9 , in that case 9 = 9*1 = 3*3 (In this case both Odd) if a+b = 9 and a-b = 1 therefore a = 5 and b = 4, thats why we have 25-16 = 9 and also if a+b = 3 and a-b = 3, a = 3 and b = 0 and thats why we 9-0 =9 so we see that only two ways we can represent the number 9 as DIFFERENCE OF SQUARES OF INTEGERS numbers. If we extend this concept. For odd Numbers, 1) Any odd prime numbers can be represented in ONLY 1 way. For example : 3 = 3*1 ONLY so only 1 way 2) Other than this any composite Odd numbers can be represented in More than 1 way

For Even Numbers: 1) Any number of the form 4K+2 CANNOT be represented at all, hence 0 ways. For example Number 6 = 2*3 = 1*6 (So we don't get any combination for either Both Even or Both Odd), hence Integer numbers IS NOT POSSIBLE. 2) Any Prime factor Multiple of 4 is ALWAYS 1 way. For example : Number 8 = 2*4 = always 1 way. The reasoning behind is for prime number 2 when mutplied with 2 we can always break up into factors of Even numbers. Hence 4*2 we can always break up. And for odd prime numbers, we already have 2 twos in 4, so also we can break up into 1 ways into both even factors. 3) Any other even numbers more than 1 way.

There are some questions where a person A and person B starts at different time and meets at a specific time and then reaches the destination at same time from where the other has started . This type of questions appear frequently in Mocks and other exams. Lets say person A and B starts from points P and Q at different times and meets at point S and reaches the points Q and P at the same time. P----------------------S----------------------Q -----------------------------------------------Time taken by person A is say t1 and speed being s1. Time taken by person B is say t2 and speed being s2. hence PS = t1*s1. SQ = t2 * s2. As they reach the destination at the same time. hence (t1 * s1)/(t2) = (t2*s2)/(t1) hence s1/s2 = root(t2/t1) This is the principle and depending upon the questions, use options to keep either the time ratio same or speed ratio same. Find the number of ways of selecting 4 books from a set of 12 books such that no 2 books are adjacent to one another. The usual method involves finding the total number of combinations and eliminating the invalid ones. Here's a little short cut.. To pick 4 books we'd require at least 7 books.. ie x-x-x-x (where x is book and - is space between books) There are 3 hpyens. subtract this from the total no. of books. Ans is 9c4 (12-3) I normally use A + B + C + D+E = 12 where + are books and A, B , C , D and E are the separators. Hence Sum of A, B, C, D, E can take 12-4 (4 Books) = 8 B, C, D >=1 and A and E can be 0. Hence Subtract 8 -3 which is 5 hence 9C4, How we can find different pythagorus triplets. The Formula is a mixed fraction n[n/(2n+1)] hence N can take values from 1,2..... hence the first mixed fraction becomes 1[1/3] = 4/3 One side of the pythagorus theorem is denominator which is 3 and other side is numerator which is 4, the hypotenuse is 4+1 = 5 Hence the triplet is 3,4,5. Lets take n = 3

2n+1 = 7 Mixed fraction becomes = 24/7 the first side becomes 7, the other side is 24 and hypotenuse is 24+1 = 25 hence 7,24,25. all the other remaining pythagorus triplet is multiples of the triplets given by the above formula n + [n/2n+1] if not mixed fraction a) In a plane if there are n points of which no three are collinear, then

The number of straight lines that can be formed by joining them is nC2. The number of triangles that can be formed by joining them is nC3. The number of polygons with k sides that can be formed by joining them is nCk.

(b) In a plane if there are n points out of which m points are collinear, then

The number of straight lines that can be formed by joining them is nC2 mC2 + 1. The number of triangles that can be formed by joining them is nC3 mC3. The number of polygons with k sides that can be formed by joining them is nCk mCk.

(c) The number of diagonals of a n sided polygon are nC2 n = n (n 3)/2. (d) The number of triangles that can be formed by joining the vertices of a n-sided polygon which has,

Exactly one side common with that of the polygon are n (n 4). Exactly two sides common with that of the polygon are n. No side common with that of the polygon are n (n 4) (n 5)/6.

Cauchy-Schwartz Equation: If a , b , c , d are four real numbers, they always satisfy the relationship (a^2+b^2)(c^2+d^2)>=(ac+bd)^2 This can be generalized to a large number of variables as (a1^2+a2^2+a3^2+.....)(b1^2+b2^2+b3^2+....)>=(a1b1 +a2b2+a3b3+....)^2 Questions: 1>Find the least value of X^2+Y^2+z^2 if X+2y+3Z=14 Sol:-->(X^2+Y^2+z^2)(1^2+2^2+3^2)>=(X*1+Y*2+z*3)^2 hence , min value= 14^2/14=14

Few Concepts: 1> For any prime number p, (p-1)times same digit is repeated , then that number formed is exactly divisible by p eg. 666666 is divisible by 7. 2>All perfect squares of the form 3K+N where N=0,1 4K+N where N=0,1 8K+N where N=0,1,4 9K+N where N=0,1,4,7 3>For any two integers satisfying, 3<a<b ---->a^b > b^A 4>If three circles touch Each other in a row and they have two direct common tangents, then their radii are in GP series 5>Product of factors of a number = (Number )^(total no of factors/2) 6>All odd natural numbers can be represented as the difference of two perfect squares. All even numbers which are multiple of 4 can be also written as the difference of two perfect squares. 7>Suppose we get a question like(I) A and B start at same time towards each other, meet at a point after time T then reach their respective destinations after time T1 and T2, or (II)their start at different time ,meet each other at a time where A takes T1 to reach and B takes T2 to reach, and then reaches their destination after time T at the same time , Then VA/VB =Root(T2/T1) And T = Root(T1*T2) where T is the same they take either to reach their meeting point in first or Destination in second case 8>If a Right Angled Triangle is rotated about its Axis to generate a cone, then the cone will have maximum volume when rotated about the smallest side as an axis and minimum when rotated about the side perpendicular to the smallest side as an axis. 9>If a n digit number is multiplied by an (n+1) digit number , the product has 2n or (2n+1) digits. 10> Difference between Principal Interest and Compound Interest for the second year=Pr^2 And Difference between PI and CI for the third year=Pr^2(r+2)where r=(R/100) and P is the amount and R is the rate of interest. 11>If A and B takes K days when working together and X+K and Y+K days when working alone respectively to complete a work then (X/K)=(K/Y) or XY=K^2. 12>The number of times the sign of the coefficients of an equation changes, gives the number of roots of a quadratic or higher degree equation. 13>The number of ways of writing any number as a sum of two or more consecutive positive numbers = number of odd factors of that number-1. Any number that doesnt have an odd factor cant be expressed

as a sum of two or more consecutive numbers. eg. 50 has two odd factors. So, it can be written in only one way as a sum of two or more consecutive positive numbers which is 11+12+13+14. Adding some more fundas: 1) Every isosceles Trapezium is a cyclic quadrilateral. 2) if in Trapezium, AB and CD are Parallel sides, then AC^2 + BD^2 = AD^2 + BC^2 + 2 * AB * CD. 3 points are required for a circle. And there are 17 points. hence 17C3 circles can be formed. But 7 points are on the same circle. To Find out Pythagorean triplets There is a formula (N2-1)/2,( N2+1/2) of any number N. For Example if the number is 3, the pythagorean triplets are (3)2-1/2 = 4 and (3)2+1/2 = 5. For any other number , i.e. !5, the triplets are (15)2-1/2 = 112 and (15)2+1/2 = 113. please solve one question from quadratic equations: if a and b(not equal to 0) are the roots of the eqation x^2+ax+b=0, then the least value of x^2+ax+b is 1)9/4 2)-9/4 3)-1/4 4)1/4 a + b = -a => a = -b/2 ab = b => a = 1 => b = -2 So, the eq becomes x^2 + x - 2 x^2 + x - 2 = (x + 1/2)^2 -9/4 => Least value will be -9/4 the min. would exist at x= -b/2*a.. and the value is = -D/4*a b is coeff of x a is coeff of x^2 hence min val = -9/4 How to find the number of factor of form (3n + 0/1/2) of number N. Its easier for the factors of kind 3n. Just find the number of factors of N/3 and that will be the answer. For factors of form (3n + 1) or (3n + 2) We know that, (3n + 1)(3n + 1) = (3k + 1) (3n + 1)(3n + 2) = (3k + 2) (3n + 2)(3n + 2) = (3k + 1) Lets consider the example of N = 3136 = (2^6)*(7^2) i) (3n + 1) We know that 2 is of form (3k + 2), but 4 is of form (3k + 1). 7 is also of form (3k + 1). So all the factors of form (3k + 1) are expressed as (2^2k)*(7^n) k van take 4 values(0, 1, 2, 3) and n can take 3 values (0, 1, 2)

=> Number of such factors = 4*3 = 12 ii) (3n + 2) We know 2 is of form (3n + 2) So, all factors of form (3n + 2) can be expressed as (2^(2k + 1))*(7^n) k can take 3 values (0, 1, 2) and b can take 2 values => Number of such factors = 3*3 = 9 Similarly, we can go for (4n + 1) or (4n + 3) Or some other form see all prime numbers (except 2 and 3) can be expressed in the form on 6k+1 or 6k-1 ,though the reverse is not true..i.e. if a number can be expressed in the form of 6k+1 or 6k-1 it is not necessarily a prime number... for example- 13=6*2+1,5=6*1-1 another way of checking whether a number is prime or not is this... i m explaining it with the help of an example.. say the number is 171 and you want check whether the number is prime or not.. so find the nearest square root of the number,in this case it would be 13(13^2=169)..so divide 171 with all prime number less than or equal to 13..in this case check if 171 is divisible by 13,11,7,5,2,3...if it divisible then the number is not prime else the number will be prime... Equations: 1) Quadratic Equation ax^2+bx+c = 0, has maxima = -D/4a at x = -b/2a [D = b^2-4ac] 2) All polynomials of degree 1 would be a straight line. hence if a area under the curve is asked, solve for x = 0 and y =0 and we can get the two points and draw straight lines. This helps in elimininating some options. 3) if the Polynomial is given as ax^n + bx^(n-1) +...+ z = 0 then sum of the roots (taken one at a time) = -b/a sum of roots taken two at a time = +c/a similarly it continues with alternate -/+ signs. 4) For Quadratic equations, if a>0, then the graph would be a parabola with opens upward and if a<0, then the graph opens downwards. There are n non-overlapping identical triangles, then at max how many bounded regions these intersecting triangle can form. 3n^2 - 3n + 1 We are drawing 8 circles on an infinite plane. How many maximum distinct regions can be obtained by drawing the 8 circles in this way? f(n) = n^2 - n + 2 the three dimensional plane is divided into distinct regions (can be unbounded) by drawing spheres. How many maximum regions can 4 spheres divide the space into? f(n) = n^2 - n + 2

Circle Division by Chords ( for n=1 to n) 1, 2, 4, 8, 16, 31, 57, 99, 163, 256 Plane Division by Lines N(n)=1/2(n^2+n+2) There are n non overlapping identical rectangles, then at max how many bounded regions these intersecting rectangles can form. 4n^2 - 4n + 1 = (2n - 1)^2 Well guys here is a small funda for calculating squares of number ending with 5. Well many may be aware of this, but this is targeted to those who are unaware of this method. So here it goes

Some quick and basic concepts 1)Rectangle, Square and Iso Trapezium can be circumscribed in a circle. Parallelogram rhombus and trapezium(not iso trapezium) can't be. 2) Area of cyclic quadrilateral = sqrt[(s-a)(s-b)(s-c)(s-d)] 3) If one pair of opposite sides are equal in a cyclic quadrilateral then the diagonals are equal. 4) Angle bisectors of cyclic quadrilateral meet and form cyclic quadrilateral. 5) In regular polygon of N sides,central angle subtended by a side = exterior angle = 360/N 6) In a regular hexagon of side A units, the longest diagonal = 2A 7) Eq. triangle of side A and an isosceles triangle 120-30-30(A being the side opp to 30 degrees) have the same area root(3)/4 * A^2 ...... and sides of 120-30-30 triangle are in the ratio 1:1:root(3)

9) If a larger cube painted by a color is broken into n smaller cubes, then no. of cubes having 3 faces painted = 8 2 faces painted = 12(n-2) 1 face painted = 6(n-2)^2 0 faces painted = (n-2)^3 find the remainder when 12^600/100 or say 34^320/107 and the like.... i've tried searching through the threads and have come across things like Chinese remainder theorem and Euler's method and stuff....but haven't been able to find a post that explains these concepts properly and how to apply them to solve the problems.... For these kind of problems you can do the following:

step1. 12^600/100 --> now since 12 is less than 100, let us find power of 12 which is nearer to 100.we get 12^2. step2. So the expr become 144^300/100 --> divide 144/100 rem = 44 step3. The expr now become 44^300/100 --> repeat step1. Like this go on and ultimately you will end with a term less than 100 in numerator. That will be the remainder. this is a bit time consuming but with practice you will get it. similarly... if numerator was 12^601, then you can write it as 12^600 * 12. Now you would have to find rem(12/100) * rem(12^600/100) In the end if numerator is greater than 100, again divide by 100. You always have to continue dividing by 100 until the numerator doesnt become less than denominator.

There are n non-overlapping identical triangles, then at max how many bounded regions these intersecting triangle can form. 3n^2 - 3n + 1

We are drawing 8 circles on an infinite plane. How many maximum distinct regions can be obtained by drawing the 8 circles in this way?

f(n) = n^2 - n + 2

the three dimensional plane is divided into distinct regions (can be unbounded) by drawing spheres. How many maximum regions can 4 spheres divide the space into? f(n) = n^2 - n + 2 Circle Division by Chords ( for n=1 to n) 1, 2, 4, 8, 16, 31, 57, 99, 163, 256 Plane Division by Lines N(n)=1/2(n^2+n+2) There are n non overlapping identical rectangles, then at max how many bounded regions these

intersecting rectangles can form. 4n^2 - 4n + 1 = (2n - 1)^2 How to find the number of factor of form (3n + 0/1/2) of number N. Its easier for the factors of kind 3n. Just find the number of factors of N/3 and that will be the answer. For factors of form (3n + 1) or (3n + 2) We know that, (3n + 1)(3n + 1) = (3k + 1) (3n + 1)(3n + 2) = (3k + 2) (3n + 2)(3n + 2) = (3k + 1) Lets consider the example of N = 3136 = (2^6)*(7^2) i) (3n + 1) We know that 2 is of form (3k + 2), but 4 is of form (3k + 1). 7 is also of form (3k + 1). So all the factors of form (3k + 1) are expressed as (2^2k)*(7^n) k van take 4 values(0, 1, 2, 3) and n can take 3 values (0, 1, 2) => Number of such factors = 4*3 = 12 ii) (3n + 2) We know 2 is of form (3n + 2) So, all factors of form (3n + 2) can be expressed as (2^(2k + 1))*(7^n) k can take 3 values (0, 1, 2) and b can take 2 values => Number of such factors = 3*3 = 9 Similarly, we can go for (4n + 1) or (4n + 3) Or some other form POWER OF THREE METHOD Q) You have 6561 balls. 1 is lighter than remaining. What is the minimum number of weighing required on a common balance to ensure that the odd ball is identified ? (a) 8 (b) 7 (c) 6 (d)5 For such questions power of 3 is used. Let there be 3 balls, 1 out of them is heavy, minimum number of weighings to find the heavy ball is = 1 First take any 2 balls, if they are in balance => the 3rd ball is heavy. if they are not in balance, we get the heavier ball.

Thus minimum number of weighings = 1 For 9 such balls, where 1 is heavy , it would be = 2 . [ In the 1st weighing 3 balls on each side of balance. if they are in balance, another weighing with remaining 3 balls is required. if they are not in balance, the heavier side having 3 balls is weighed again.] Thus minimum number of weighings = 2 Similarly for 10-27 balls, minimum number of weighings = 3 28 - 81 balls, minimum number of weighings = 4 82 - 243 balls,minimum number of weighings = 5 244 - 729 balls, minimum number of weighings = 6 730 - 2187 balls, minimum number of weighings= 7 2188 - 6561 balls, minimum number of weighings = 8 Therefore, answer would be (a)8. Ex : For 425 balls, minimum number of weighings = 6 (Since 425 lies in the range 244 to 729.) Basically, power of 3 can be summarized as : 3^n = Number of balls where n = minimum number of weighings on a common balance.

N^4321 % 4 2323...23 =3*odd+3*odd+...50 times=even So, 2323...23^anything would give remainder 0 with 4. 2) 125=3k-1, 25=3k+1,5=3k-1,1=3k+1 4213=3k-1+ 3k+2 +3k-1=3k 1423=3k-1+3k+1+3k+1=3k+1 1243=3k-1+3k+2+3k-1=3k 3421=3k+3k+1+3k+1+3k+1=3k So, (1423)^2143 is not oven. So, option (b) How many right angled triangle can be formed of integral side such that one of the three sides is 84 ? x^2 + 84^2 = y^2, where 84 is not hypotenuse x^2 - y^2 = 84^2 (x + y)(x - y) = 4*(4*36*49) So, 13 ways 84 can never be hypotenuse of triangle having integral sides So, 13 ways

Consider all quadrilateral PQRSs such that PQ=14, QR=9, RS=7, and SP=12. What is the radius of the largest possible circle that fits inside or on the boundary of such a quadrilateral? Suppose radius = r, then area = (1/2)r(14 + 9 + 7 + 12) = 21r Also, area is maximum when its a cyclic quad => 21*r(max) = (7*14*9*12) = 426 => r(max) = 26

In a surprising pattern of an entrance test, there were 10 questions, each having two parts a and b. Find all the possible number of selections a student can make for answering one or more questions in the paper. (Note: The student can answer a question by attempting at least one out of part a or part b of the question.)

Its 4^10 - 1 For every question we have three options:i) Attempt part a ii) Attempt part b iii) Attempt both iv) Leave the question So, 4^10 ways, but we need to remove the case when no question is attempted. Hence, 4^10 - 1 ways

5 couples stand in a line. how many ways can they be arranged such that exactly one couple is togethr? in how many steps can i break a tile of m*n dimension into pieces of 1*1. assuming i can break only one tile at a time? It can have at most 5 digits as 7^5 > 5^6 So, it will be 5^5 - 1, i.e, 3125 is the largest such number 2) I think its 5*2*9! - 2{C(5, 2)*4*8! - C(5, 3)*8*7! + C(5, 4)*16*6! - 32*5!} = 1102080 (principle of inclusion-exclusion) 3) Is it mn - 1???

131. Let a, b, c, d be four integers such that a+b+c+d = 4m+1 where m is a positive integer. Given m, which one of the following is necessarily true? 1. The minimum possible value of a2 + b2 + c2 + d2 is 4m22m+1 2. The minimum possible value of a2 + b2 + c2 + d2 is 4m2+2m+1 3. The maximum possible value of a2 + b2 + c2 + d2 is 4m22m+1 4. The maximum possible value of a2 + b2 + c2 + d2 is 4m2+2m+1

Cauchy Schwartz kehta hai ki (y kya hai?) Hw to implement? (p^2+q^2+r^2+s^2)(a^2+b^2+c^2+d^2)>=(pa+qb+rc+sd)^ 2

square the equation a+b+c+d = 4m+1 you will get a2 + b2 + c2 + d2 +2((ab + ac + ad + bc + bd + cd)=(4m+1)^2 a2 + b2 + c2 + d2 will be minimum when ab + ac + ad + bc + bd + cd is maximum and that happens for a=b=c=d=m + 1/4 so 2((ab + ac + ad + bc + bd + cd) = 12(m + 0.25)^2= 12m^2 + 6m + .75

calculate a^2+b^2+c^2+d^2

it comes out to (4m+1)^2 - 12m^2 + 6m + .75 which is 4m^2+2m + .25 since m is an integer, i will go for option 2

14)A quadratic with integral coefficients has two distinct positive integers as roots, the sum of its coefficients is prime and it takes the value -55 for some integer. The sum of the roots is (a) 32 (b) 20 (c) 24 (d) none of the these
Find the number of 3 digit numbers which when divided by 11 leave a remainder of 9 and when divided by 7 leave a remainder of 2? a) 11 b) 12 c) 14 d) 16 c) Given |p| >= 1. What is the area of the region bounded by the lines |x| = |p|, x-y = 0 and the x-axis? 1) 2) 3) 4) 2p^2 p^2 p^2/2 p^2 + 1

Two distinct numbers are chosen at random from the set of first 30 natural numbers. The probability that a2 - b2 is divisible by 3 is ?? squares are of the form 3n or 3n+1 no. of the form 3n give perfect squares of the form 3k others give perfect squares of the form 3k+1 For div. by 3, either both should be 3k, i.e. 1/9 or both should be 3k+1/3k-1=4/9 5/9

Balu can reach A,C,G,I and Kalu can reach all of A,C,G,I,E. So they can meet only at A,C,G,I. Balu can move to 4 possible vertices in 8 ways, and kalu can move to A,G in 3 ways, E in 2 ways, and C and I in 1 way. So, there are 10 ways for Kalu too. When Lalu moves to A, Balu and Kalu can meet at: C:Probability=1/4 * 1/10=1/40 G:Probability=1/4 * 3/10=3/40 I: Probability=1/4* 1/10=1/40 Probability: 5/40 When Lalu moves to G, symmetrical cases would be there. Probability: 5/40 When Lalu moves to E, the case of them meeting at both A and G would also be a possibility. Probability: 8/40 Total probability: 1/3 *18/40=3/20 There are exactly 1000 tokens in a big box which are numbered from 1 to 1000. There is a group of 30 students who have taken the tokens one after the other in the following way. The first student took all the tokens which were numbered with multiples of 2, the second student took all the tokens that are multiples of 3 and so on. Finally, the 30th student takes the tokens that are numbered with multiples of 31. a) How many tokens did the 10th student take?? b) How many students took minimum (same) number of tokens?

c) It is given that there are 168 prime numbers between 1 and 1000, then how many tokens are still left in the box. a) 11 and all the products of primes greater than 11. 1+all primes from 11 to 90. b) all non-primes till 31. 19 c) 168-{2,3,5,7,11,13,17,19,23,29,31}=157 plus the first token. 158 in all in which base is 232 a perfect square 2b^2+3b+2 is a perfect square. 7 aa gaya.

If 1/6 of 30 is 4, what is 1/4 of 10? Means 30 is 6*4 =24 As (3b)/6=4 3b=24 b=8. base 8 ki baat ho rahi hai! 10 is 8. 1/4 of 8 is 2.

if the integers m and n are chosen from 1 to 100 randomly then the probability that a no. of the form 7^m + 7^n is divisible by 5 is ?? euler(5)=4 2^(m%4) +2^(n%4) should be a multiple of 5. for 5: [4,1]: m must be 4k+2, n must be 4k or vice versa: for 10: [8,2]: m must be 4k+3, n must be 4k+1 or vice versa: Probabilty: 2/16+2/16=1/4 Let , be positive real numbers such that + = 1, then find the maximum possible value of [(^3 + ^3) (^4 + ^4)] (1) 1/2 (2) 1/4

(3) 1/8 (4) 1/3 (5) 1/6 put them as close as possible. put a=b=1/2 value is 4/16-2/16=2/16=1/8 This is either the minimum or the maximum. now put them as far apart as possible! Check at a=1, b=0.(since they are real, b can be 0.000001 i.e. slightly greater than 0) value is 0. So, 1/8 is the maxima

Which of the following rational numbers lies between (1/5)^6 and (1/5)^9 1) (1/7)^8 2) (1/7)^7 3) (1/6)^7 4) (1/4)^6 5) None of these
log2=0.3010 log3=0.4771 log(1/5)^6=6*log(0.30-1)=-6*0.7=-4.2 log(1/5)^9=-9*0.7=-6.3 log6=0.30+0.47=0.77 -7*0.77=-5.39 Option (3) hona chahiye. What is the remainder on dividing 6^83 + 8^83 by 49? euler(49)=42 anything^84 would give remainder 1 with 49. (6^43)*6 gives remainder 1, i.e. p*6=49k+1 ; 49k=6p-1 49 is 6k+1, we need to multiply it with a no. of the form 6k-1; So k=5.

6p=246; p=41 (8^43)*8 gives remainder 1, i.e. 8p=49k+1; 49k=8p-1. 49 is 8k+1, we need to multiply it by a no. of the form 8k-1; so k=7. p=43 remainder is (41+43)%49=35 8888888888888......A9999.... is divisibile by 7.. both 8 and 9 are in 50 in number.. what is the value of A?? Divisibility rule for 7 is same as that of 1001, i.e picking up triplets and adding and subtracting them alternately. 48 on either side will get cancelled. We're left with 88A99. 88A99 should be div. by 7. A is 5. In a class of fifty five students each one has either opted for Computer or Hindi as a subject and was either interested in debate or quiz. The number of students who have opted for Computer and are interested in debate is two-third of the number of students who have opted for Computer and are interested in quiz. Similarly the students interested in Hindi are twice the number of the students interested in Computer and quiz. Find the total number of students who have opted for Hindi. OPTIONS 1) 30 2) 15 3) 25 4) 20 5) Cannot be determined ___debate--->a Hindi---___quizzing--->b ___debate--->c Computer---___quizzing--->d a+b+c+d=55 3c=2d a+b=2d

3d+2d/3=55 11d/3=55 d=15 a+b=30 30 have opted for hindi. Which is the smallest number which can form the side of 10 distinct right triangles with integer side lengths? N=2^8*3^10*5^8*7^2..What is number of factors of this numner N which are ""multiples"" of 360 but not 540? 360=2^3*3^2*5 540=2^2*3^3*5 exponent of 3 should be 2 ; because if it's 3, the number will become a multiple of 540(as 540 has 3 powers of 3), which we don't want. Now that we've ensured that the number will not be a multiple of 540, we can include all the available exponents of 2,5,7 in the given number. exponent of 2 can be from [3-8], minimum has to be 3, so that it's a multiple of 360(360 has 3 powers of 2), maximum is 8, as specified in the number. exponent of 5 from [1-8] exponent of 7 can be [0-2] Total: 1*6*8*3=144 P and Q are natural numbers. When P is divided by Q, the remainder left is 13. When P is divided by 9Q, the remainder left is 49. Find the remainder when P is divided by 3Q 1) 2) 3) 4) 7 13 49 cannot be determined

When P is divided by Q, the remainder left is 13. => P = Qx + 13 When P is divided by 9Q, the remainder left is 49. => P = 9Q*y + 49 So, Qx + 13 = 9Qy + 49 => Q(x - 9y) = 36 As left side is divisible by Q; right side should also be divisible by Q.

So, Q is a factor of 36. So, Q can be 18 or 36. Q cannot be 12 as P leaves remainder by Q as 13. So, Q > 13. So, 3Q is 18*3 or 36*3. Both are greater than 49. Now, we already have, P = 9Q*y + 49 => P = 3Q*(3y) + 49 Now we need to find out remainder when P divided by 3Q. Means 9Qy+49 divided by 3Q will exactly divide first part that is 9Qy so remaining is 49 only. So, for remainder we can ignore 3Q*3y as it will be divisible by #q. So, we are left with 49. And as 3Q > 49; remainder of 49 with 3Q = 49 Let there be Z number of rational triplets (g, h, i) for which g, h, i are the roots of x^3 + gx^2 + hx + i = 0. Then which of the following is the value of Z ? (a)0 (b)1 (c)2 (d)3 (e) Cannot be determined what is the remainder when 12341234.....400 terms is divided be 909?? 909=9*101 remainder by 1000%9=1 For 101, we pick up pairs of numbers from the right alternately. 34+34++...+34-12-12-..-12=22*100=2200%101 101k+79, and 9k+1 So, it's 685 Find the number of non-negative integral solutions for the equation 3a + 4b + 12c = 432 ? 3a+12c=432-4b 3(a+4c)=4(108-b) a should be a multiple of 4, b should be a multiple of 3. let a=4x,b=3y (x+c)=(36-y) x+y+c=36 So 38C2 soltutions honge! BASE System:If a number abcd is mentioned in base n; then; abcd = d + c*n + b*n^2 + a*n^3 and so on

And of course a, b, c and d < n In base 10 system, we use 10 digits from 0 to 9. In base 9 system, we use 9 digits form 0 to 8. In base 5 system, we use 5 digits from 0 to 4. Now, in which base is 232 a perfect square It 232 in base n = 2 + 3*n + 2*n^2 Now, this number should be perfect sqaure. try options. That becomes easy now.. With 7; you will get 2+21+98 = 121 = 11^2 With n=2 also you get number = 16 But it base 2 system; we will use digit from 0 to 1. So, digits 2, 3 cannot be used. So, start trying option with value greater than 3. Now, 1/6 of 30 is 4. So, 6*4 = 30 In base 10; 6*4 = 24.. Now, if in base n systen; if you get it more than 24; it means base n is less than 10. So, we can try for 9 or 8 or 7. It cannot be 6 as 6 is one of the digits. See.. 101 = 101 in base 10 But, 101 = 5 in base 2 So, as base increased, number appears smaller.

A decimal no. which is represented by the radix of 3,4,5 and 7 has 1,2,3 and 5 as the digits to its extreme right. the smallest such positive integer is ? Number is of the form: 3k+1,4k+2,5k+3,7k+5 i.e. 12k+10, 35k+33 i.e. 418 Last digit of a number in any base gives the remainder of the number by that base. Jaise 57 in base 10 gives remainder 7 with 10. Proof: Assume the base is x, and the number be abcd The number, when expressed in base 10 is a.x^3+b.x^2+c.x+d If we find the remainder of this number by x, then only the last digit would contribute as all other terms are a multiple of the base x.

In the given question, the number gives remainder 1 with 3, 2 with 4, 3 with 5 and 5 with 7. Hence the result.

F(x) = p(x) + (3x+4) Lets assume p(x) a polynomial eqn of 6 degree which gives 0 for f(1) up to f(6) So, can we take p(X) = (x-1)*(x-2)*(x-3)*(x-4)*(x-5)*(x-6) it will be 0 for x=1 to x=6 f(x) = (x-1)*(x-2)*(x-3)*(x-4)*(x-5)*(x-6) + (3x+4) so f(7) = (7-1)*(7-2)*(7-3)*(7-4)*(7-5)*(7-6) + (3*7 + 4) f(x) f(x) f(x) f(x) f(x) f(x) 7 = (x - 1)p(x) 10 = (x - 2)q(x) 13 = (x - 3)r(x) 16 = (x - 4)s(x) 19 = (x - 5)t(x) 22 = (x - 6)n(x)

We can see that polynomial f(x) - (3x - 4) has 6 roots, 1, 2, 3, 4, 5, 6 => f(7) = (7 - 1)(7 - 2)(7 - 3)(7 - 4)(7 - 5)(7 - 6) + (3*7 + 4) = 745

Funda 1 For checking divisibility by p, which is of the format of 10 n 1, sum of blocks of size n needs to be checked (blocks should be considered from the least significant digit, or the right side). If the sum is divisible by p, then the number is divisible by p. Example 1, Check if a number (N = abcdefgh) is divisible by 9 9 is 101 1 Sum of digits is done 1 at a time = a + b + c + d + e + f + g + h = X

If X is divisible by 9, N is divisible by 9 Also, N is divisible by all factors of 9. Hence the same test works for 3.

Example 2, Check if a number (N = abcdefgh) is divisible by 99 99 is 102 1 Sum of digits is done 2 at a time = ab + cd + ef + gh = X If X is divisible by 99, N is divisible by 99 Also, N is divisible by all factors of 99. Hence the same test works for 9, 11 and others.

Example 3, Check if a number (N = abcdefgh) is divisible by 999 999 is 103 1 Sum of digits is done 3 at a time = ab + cde + fgh = X If X is divisible by 999, N is divisible by 999 Also, N is divisible by all factors of 999. Hence the same test works for 27, 37 and others.

Funda 2 For checking divisibility by p, which is of the format of 10 n + 1, alternating sum of blocks of size n needs to be checked (blocks should be considered from the least significant digit, or the right side). If the alternating sum is divisible by p, then the number is divisible by p. (Alternating Sum: Sum of a given set of numbers with alternating + and signs. Since we are using it to just check the divisibility, the order in which + and signs are used is of no importance.) Example 1, Check if a number (N = abcdefgh) is divisible by 11 11 is 101 + 1 Alternating sum of digits is done 1 at a time = a b + c d + e f + g h = X If X is divisible by 11, N is divisible by 11

Example 2, Check if a number (N = abcdefgh) is divisible by 101 101 is 102 + 1 Alternating sum of digits is done 2 at a time = ab cd + ef gh = X If X is divisible by 101, N is divisible by 101

Example 3, Check if a number (N = abcdefgh) is divisible by 1001 1001 is 103 + 1 Sum of digits is done 3 at a time = ab cde + fgh = X If X is divisible by 1001, N is divisible by 1001 Also, N is divisible by all factors of 1001. Hence the same test works for 7, 11, 13 and others.

Funda 3: Osculator/seed number method For checking divisibility by p, Step 1: Figure out an equation such that p n = 10m 1 If we have this equation, the osculator/seed number for p will be -+ m. (-m in case of 10m+1 and +m in case of 10m 1) Step 2: Remove the last digit and multiply it with the seed number. Step 3: Add the product with the number that is left after removing the last digit. Step 4: Repeat Steps 2 and 3 till you get to a number which you can easily check that whether or not it is divisible by p.

Example, Check whether 131537 is divisible by 19 or not. 191 = 102 1 (Seed number is +2) 131537 13153+72=131671316+72=1330133+02=133 133 is divisible by 19 131537 is divisible by 19

Find the least number N which when divided by P successively 3 times, leaves remainders of 2, 3, 7 respectively, where P is the least number such that when it is divided by 7, 6 and 5 it leaves remainders of 2, 3, 4 respectively. first step is to find P Now, P = 7x+2 = 6y+3 = 5z+4 => P = 9 Now, N leaves remainder of 7, 3 and 2 when successively divided by P. For smallest number: We can get N like: First, 7 ...................... Remainder by P is 7 Then 7*9 + 3 = 66 ...... Remainder by P is 3 Then 66*9 + 2 = 596 ... Remainder by P is 2 So, in general: We can take numbers like: a = ((9x+7)*9) + 3) * 9) + 2 When x = 0; we get 596 So, in general: We can take numbers like: a = ((9x+7)*9) + 3) * 9) + 2 When x = 0; we get 596 Now, you just need to try maximum values of x like: (9000-2) / 9 => Then (-3) and divide by and then (-7) and divide by 9. You will get some approximate value. An integer less that will be higest value for x. So, x can take values from 0 till that higest value..

How many numbers below 100 are co-prime to 100 but not co-prime to 1500? (a) 12 (b) 14 (c) 16 (d) 18 100 = 2^2 * 5^2 So, number of co-primes to 100 below 100 = 100 *(1/2) *(4/5) .... standard formula = 40 (3,7,9,11,13,17.........) Now, these 40 numbers are odd numbers which are not multiples of 5. 1500 = 3 * 5* 100 => 3 is the key as it is not factor of 100. So, out of those 40, 3 will have some factors which are not co-prime to 1500.

Now, till 100; 3 has 33 multiples of which 17 are odd. So, these 17 numbers are not co-prime to 1500. (3,9,15,21,27..........)

But, these also cotain multiples of 3*5 i.e 15. They are 15, 45, 75. These are co-prime to 100 as well. So, co-primes to 100 but not to 1500; which are less than 100 are: 17-3 = 14 question: how many of the first 2100 natural numbers are either prime to 6 or to 15 ?? Coprime to 6 = 2100(1/2)(2/3) = 700 Coprime to 15 = 2100(2/3)(4/5) = 1120 Coprime to 35 = 2100(4/5)(6/7) = 1440 Coprime to 30 = 560 Coprime to 105 = 960 Coprime to 210 = 480 => Number either prime to 6 or 15 or 35 = 700 + 1120 + 1440 - 2((560 - 480) + (960 - 480)) - 3*480 = 700

Q)In an integer d1d2...dk from left satisfy di<di+1 for i odd and di>di+1 for i even. How many integers from 1000 to 9999 have four distinct digits? When 0 is not there and the 4 digits are a, b, c, d such that:a > b > c > d, then Possibilities are cadb, cbda, dacb, dbca and badc So, C(9, 4)*5 = 630 numbers When 0 is there and other three are a, b, c, (a > b > c), then Possibilities are ca0b, cb0a, ba0c So, C(9, 3)*3 = 252 numbers Total = 882 A Wooden rod AB is 1 Mtr long. Raju makes a mark every 3 cm along the rod starting from end A . Vikram makes a mark every 7cm and Aditya every 9 cm both starting from end B. If the rod is cut at each of the points where there is a mark , then how many pieces would it be cut into ? Series 1,10,19,....... 3,6,9........... 2,9,16,23............. Cuts are made at positions 3k, 7k + 2 and 9k + 1 3k -> 33 cuts 7k + 2 -> 14 cuts 9k + 1 -> 11 cuts 3k = 7n + 2 21p + 9, 5 common terms 7k + 2 = 9n + 1 63p + 37, 1 common term

So, 33 + 14 + 11 - 5 - 1 = 52 cuts => 53 pieces

1/1*5 + 1/5*9 + 1/9*13 + ...........+ 1/221*225 a. 28/221 b. 56/221 c. 56/225 d. none of these 1/(1*5) + 1/(5*9) + .... + 1/(221*225) =1/4 * [(1/1 - 1/5) + (1/5 - 1/9) + ....... + (1/221 - 1/225)] ........ This is because (1/1 - 1/5) = 4/(1*5) and same approach for other terms. So, all therms in bracket, except first one and last one, will cancel out. So, sum = 1/4 * [1/1 - 1/225] = 56/225 1/1 + 1/3 + 1/6 + 1/10 + 1/15..... a.2 b.2.25 c.3 d.4 It should be 2. S = 1/1 + 1/3 + 1/6 + 1/10 + 1/15 + ..... = 1/1 + 1/(1+2) + 1/(1+2+3) + ... nth term = 2/(n(n+1)) = 2[1/n - 1/(n+1)] => S = 2 [1/1 - 1/2 + 1/2 - 1/3 + 1/3 - 1/4 + 1/4 - 1/5 + 1/5 - 1/6 + .....] = 2 [1] =2 s=1/1+1/3+1/6+1/10+1/15 = 1+1/3(1+1/2)+1/5(1/2+1/3)+1/7(1/3+1/4)......... = 1+1/3(3/2)+1/5(5/6)+1/7(7/12)......... = 1+1/2+1/6+1/12......... = 1+1=2

256! is expanded and expressed in base 576 . how many zeroes will this expression have on its right end?

Such questions are done like this: In base 10 system, 1000 has 3 zeros; because it has 10^3 as a factors. 300 has two zeros because, it has 10^2 as a factor. So, in a syatem with base n, number of zeros will be equal to maximum factor n^x for that number. So, we need to find out what is maximum of (576)^x in 256!. Now, 576 = 2^6 * 3^2 In 256!; we have 2s = 128+64+...+1 = 257 And 3s = 85+28+9+3+1 = 126 So, maximum power of 2^6 = 257/6 => 42.xx => 42 And maximum power of 3^2 => 126/2 = 63 So, we have 42 as limiting power pf 576. So, 576^42 is a factor of 256!. So, there will 42 zeros at the end of 256!.

What is the total numbr of zeros in 23! in base 7 understand when do we get a zero in our decimal system? When the number equals the base system number, 10. Then we further see that 10=2*5, since 5s are less, we find the nuber of 5s. Now, no of zeroes in Base 7, would be the number of 7s occuring in the number. In this case of 23!, there are 3 7s and hence your answer!

For positive integers n, denote by D(n) the number of pairs of different adjacent digits in the binary (base two) representation of n. For example, D(3) = D(112) = 0, D(21) = D(101012) = 4, and D(97) = D(11000012) = 2. For how many positive integers n less than or equal to 97 does D(n) = 2? (A) 16 (B) 20 (C) 26 (D) 30 (E) 35

For D(n) to be 2; we need first and last digit 1 for sure. Examples: 101, 1001, 1101 etc. So, we get D(n) = 2 with nunimum 3 digit numbers: For 3 digits: 101 => Only 1 case For 4 digits: 1001 => 3 cases ... now these 3 cases are 1001, 1101 and 1011 If you see.. 1001 will be one case Then, 1011 => If we place 1 at that bold digit, we will get cases equal to cases of 3-digit numbers. As last 2 digit can can be treated as single 1. And then, we will 3rd case of 1101... we will get such cases equal to (maximum number of zeros in between - 1) So, fopr 4 digits, it is 3 Using same logic, for 5 digits; it will be 1 + f(4) + (5-3) = 1+3+2 = 6 For six digits = 1 + f(5) + (6-3) = 1+6+3 = 10 For 7 digits, 1 + f(6) + (7-3) = 1+10+4 = 15 We can have maximum of 7 digit numbers. So, total numbers = f(3) + f(4) + f(5) + f(6) + f(7) = 35

(ax^3 + 12x + 8 ) has a factor of the form (x^2 + px + 1). Find the sum of all of the possible values of a.
(x^2 + px + 1) => product of roots = 1 (ax^3 + 12x + 8 ) ==> product of roots = -8/a that means -8/a is one of the roots , now put this root in initial eqns "(ax^3 + 12x + 8 ) " now it will be a quadratic in terms of 'a' . just need to look sum of roots. that is 12 Cauchy-Schwartz Equation: If a , b , c , d are four real numbers, they always satisfy the relationship (a^2+b^2)(c^2+d^2)>=(ac+bd)^2 This can be generalized to a large number of variables as (a1^2+a2^2+a3^2+.....)(b1^2+b2^2+b3^2+....)>=(a1b1 +a2b2+a3b3+....)^2 Questions: 1>Find the least value of X^2+Y^2+z^2 if X+2y+3Z=14 Sol:-->(X^2+Y^2+z^2)(1^2+2^2+3^2)>=(X*1+Y*2+z*3)^2 hence , min value= 14^2/14=14

A shopkeeper in his daily sales has to weigh all whole numbers of kilograms up to 35 kg with a hand common balance. What is the minimum number of standard weights he should have? (1) 4 (2) 5 (3) 6 (4) 3

If it is common balance, it should be 4. We need all powers of 3. 1, 3, 9, and 27. Using this, we can form all numbers till 3^4 - 1 = 80 1=1 2 = 3-1 3=3 4 = 3+1 5 = 9 - (3+1) 6=9-3 7=9-3+1 8=9-1 9=9 and so on... What is the number of 3 digit perfect squares that exist in base 5 system? a)5 b)6 c)7 d)8

Here, as number has 3 digits in base 5; we need to find all sqaures between 5^2 and (5^3 - 1) i.e 25 and 124. So, from 5^2 to 11^2 => 7 such numbers. How many integral values of x are possible for which (2^70 + 2^1039 + 2^x) equals the square of a whole number?

Q!!)What is the remainder when 99008800......1100551 is divided by 440?

990088007700660055004400330022001100000 is divisible by 11 and 40 => Remainder will be given by remainder when 551 (last three terms) is divided by 440 => 111 is remainder

AB= 9C + 1 where A,B,C are natural and 100<=A<=200. How many different values can A take? 1. 101 2. 50 3. 11 4. None

AB divided by 9 leaves remainder of 1. So, A can cabe of forms 9x+1 when B is of form 9x+2. A can cabe of forms 9x+2 when B is of form 9x+5. And so on.. Basically, A should not be a multiple of 3 as right side is not divisible by 3. So, all values except multiples of 3. => 101 - 33 = 68 such numbers.

4x - Ay = B has a number of integral solutions. HCF(A,4) = 1 and the number of solutions for (x,y) for 0<x,y<500 is 45, the number of possible values of A 1.3 2.2 3.1 4.None of these 4x = Ay + B .. As 4 and A are co-primes, A is odd. So, A = 1, 3, 5, 7... When A = 1; B should leave remainder by 4 as -1. And then y can be 1, 5, 9, ... We will continue till 45th value which is 187. And then adust B such that 46th value makes x more than 499. Same thing we can do for 3. Here, B will leave remainder by 4 as 1. And x willbe from 1 to 187. We can do it for 5, 7 as well as 9. But, we cannot do it for A = 11; as 11*187 > 2000 => x will go beyond 500 if B is +ve. And if B is -ve, x will become less than 0. So, condition gets violated. So, A can be 1, 3, 5, 7 and 9. Total 5 values. We cannot take A as -ve because -ve numbers are not considered co-primes. Base System:-

in which base is 232 a perfect square

2b^2+3b+2 is a perfect square. 7 aa gaya.

If 1/6 of 30 is 4, what is 1/4 of 10? (3b)/6=4 3b=24

b=8. base 8 ki baat ho rahi hai! 10 is 8. 1/4 of 8 is 2. base system is like this.. If a number abcd is mentioned in base n; then; abcd = d + c*n + b*n^2 + a*n^3 and so on.. And of course a, b, c and d < n In base 10 system, we use 10 digits from 0 to 9. In base 9 system, we use 9 digits form 0 to 8. In base 5 system, we use 5 digits from 0 to 4. Now, in which base is 232 a perfect square It 232 in base n = 2 + 3*n + 2*n^2 Now, this number should be perfect sqaure. try options. That becomes easy now.. With 7; you will get 2+21+98 = 121 = 11^2 With n=2 also you get number = 16 But it base 2 system; we will use digit from 0 to 1. So, digits 2, 3 cannot be used. So, start trying option with value greater than 3. Now, 1/6 of 30 is 4. So, 6*4 = 30 In base 10; 6*4 = 24.. Now, if in base n systen; if you get it more than 24; it means base n is less than 10. So, we can try for 9 or 8 or 7. It cannot be 6 as 6 is one of the digits. See.. 101 = 101 in base 10 But, 101 = 5 in base 2 So, as base increased, number appears smaller..

You might also like